<<

Ministry of Public Heals Service of Ukraine «Ukrainian Medical Stomatological Academy»

«APPROVING» on the sitting of chair of obstetrics and gynecology №1 of UMSA (protocol №1 from 28. 08. 2020)

Acting manager of chair of obstetric and gynecology №1 professor ______A.M. Gromova

METHODICAL POINTING for the independent work of students for preparation to practical lesson

Educational Obstetric and gynecology subject Modul № 3 Subject of Benign tumors of the female genital organs. lesson Dyshormonal of the breast. Surgical interventions in gynecology Preparation and postoperational management of gynecological patients in urgent and planned operations. Prevention of HIV Course VI faculty International faculty (specialty “medicine")

Poltava – 2020 Benign tumors of the female genital organs. Dyshormonal diseases of the breast. Surgical interventions in gynecology Preparation and postoperational management of gynecological patients in urgent and planned operations. Prevention of HIV

Benign tumors of women’s reproductive organs.

UTERINE Uterine myoma (fibromyoma, ) — is a benign tumour which contains varying amounts of muscle and fibrous elements. Concerning gynecologic diseases benign tumours are found in 10-25% of all the cases, although during the last years the tendency of increasing the quantity of these tumours is observed. The myoma arises seldom in young women. The risk of grows after 35-40 years, at the age which is close to climacterium. Later beginning of menstrual function, irregular menstrual cycle, high frequency of induced abortions are present in the past history of the patients. Therefore, 35-40 years women are patients at risk for uterine fibromyoma. Tumour histogenesis and structure. Uterine myoma belongs to tumors, which are growing from mesenchyma. It has three consecutive stages in its morphogenesis. They are:  active region of growth formation  growing of tumor without differentiation  growing of tumor with differentiation and maturation The areas of growth are formed mainly around the vessels. These regions are characterized by a high level of metabolism and increased capilary and tissual permeability which stimulate the tumor growing. has in its development parenchymal-stromal features of that layer, from which it has been educed, therefore the parenchyma and stroma ratio in a tumor is different. Leiomyoma is developed at predominance of muscle elements, in the structure of fibromyoma fibrous tissue is predominant. The consistency of tumor depends on fibrous and muscle tissue ratio: the more there are muscle fibers, the more the tumor is mild at . Myomas are classified according to histologic structure as myoma, fibromyoma, angiomyoma and . According to the speed of growing there are the tumors which are growing slowly and quickly. According to histogenesis peculiarities there are distinguished simple and proliferative myomas. Proliferative myomas contain much more atypical muscle elements, where is a great number of plasmatic and lymphoid cells and increased mitotic activity. The incidence of proliferative myomas happen twice more often in the patients with fast growing tumors. Very often uterine fibroids arise in places of complex interlacing of muscle fibers of — near tubal angles, on uterine center line. The myoma is charac- terized by the effusive growing. As compared with fibroids they move apart tissue without destroying it. Tumor is growing simultaneously with tissue mass surrounding it. Uterine fibroids have few veins, basic amount of vessels is situated in pseudo-capsule. Uterine fibroids' lymphatic system is atypical without absorbent vessels. Uterine fibroids are deprived of nervous terminals, choline and adrenergic nervous frames. According to their location in the uterus myomas are classified into:  subserosal — subperitoneal uterine fibroids, which are growing under the outer serosal layer of the uterus, may have a wide or thin pedicle.  interstitial (intramural, intraparietal) — uterine fibroids, which are growing within the muscular wall of the uterus  submucosal — uterine fibroids which are growing under the uterine mucous into the  atypical forms of uterine fibroids location: retrocervical myoma grows from the posterior surface of the uterine , it is situated within a retrocervical fat; paracervical myoma grows from the lateral part of uterine cervix, it is situated in the paracervical fat; intraligamentary myoma grows from the uterine body or cervix within the broad ligaments. The fibromyoma can have one fibroid (nodulosus fibromyoma), many fibroids (multiple fibromyoma) and diffuse growth (diffuse fibromyoma). Hormonal status of the patients with fibromyoma. They are considered hormonally depend tumors because the growth of these tumors is related to production. In the majority of cases these patients have an hormonal dysfunction of which is characterised by anovulatory cycles, corpus luteum insufficiency. It leads to hyperestrogenemia and lowering of level. Small cystic changes in ovaries occur due to hormonal disordes. Uterine and are under the influence of estrogenic . Their excessive amount in blood can lead to endometrial hyperplastic processes and cystic changes in myometrium. Such local hyperhormonemia leads to pathological hypertrophy of myometrium. Not only sexual hormones synthesis, metabolism and interaction impairment, but also the state of the myometrial receptors especially large activity of the estrogen receptors as compared with receptors, take part in a pathogenesis of uterine fibromyoma. Fibromyoma grows slowly without any proliferative changes at presence of small cystic changes in ovaries with nonsignificant hyperestrogenemia. Fibromyoma growing depends on its type, location, blood supply and patient's age. Fibromyoma grows quickly in young patients, particularly during , as the fetoplacental complex synthesizes large amount of estrogenic hormones, which are tumor stimulating growing factor. Quite often fibromyoma accelerates its growing in climacterium, when there is a rearrangement of woman's hormonal system. Ovaries undergo polycystic degeneration at that time. Clinic. Clinical manifestation of fibromyomas depends on uterine fibroid's location, size of tumour, rate of its growing, and also presence of complications. Of the most myomas there are not any symptoms at the initial stages. The main symptoms are pain, bleeding, sensation of pelvic heaviness in the lower part of the abdomen, progressive increase in pelvic pressure, , , pressure on the rectum. These symptoms most commonly occur during the excessive growth of tumor, and sometimes they testify development of secondary degenerative or inflammatory changes in fibromyoma tissue. Menstrual function in the patients does not variate in case if tumor is sub-serosal because attached to the uterus by only a stalk or on a wide basis under a peritoneal integument and it is practically outside of uterine borders. Another spectrum presentation includes patients with atypical (subperitoneal) location of uterine fibroids: the tumors from the anterior wall of the uterus and antecervical location can press upon and cause dysuric signs; pressure on the ureters (as they traverse the pelvic brim) leads to hydroureter and sometimes to . Retrocervical location of uterine fibroid due to intensive growing can spread in all small pelvic, compressing rectum and provoking . Intraligamentary tumor during its growing moves apart the broad ligament of the uterus. As the ureters are passing in the lower areas of parametrium, the tumor results in pressure upon ureters leading to hydroureters or hydronephrosis. Cyclic is present but it is painful (algomenorrhea). Submucosal location of uterine fibroid is characterized by cramping cyclic menorrhagia which has been changed into acyclic bleeding. Monthly appreciable bleeding leads to the secondary deficiency . Characteristic dystrophical myocardial changes called "myom' heart" result from the secondary anemia and chronic hypoxia and are often found in patients with fibromyoma. Liver function is frequently broken in these patients. Probably, these changes are the result of steroid hormones metabolism dysfunction. Hypertrophy of the left ventricle, myocardial dystrophy, ischemic heart disease, idiopathic arterial are also present in these patients. In most of the patients after fibromyoma removal the arterial pressure is reduced to the normal level. This fact confirms the idea of pathogenetic connection of fibromyoma with changes in myocardium and rising of arterial pressure. Diagnosis. History of the patients includes hereditary predilection (myoma in mother and other reproductive organs tumors in close relatives); menstrual dysfunction, late beginning of menarche and metabolism infringement (, mellitus). Reproductive dysfunction (infertility, pregnancy loss), induced abortions (mucous and myometrium trauma should lead to endometrial receptor device changes), extragenital diseases, which caused endocrine and ovarian disordes, in particular can be present in these patients. Bimanual examination in uterine fibromyoma has characteristic signs. It includes the presence of a large midline mobile pelvic mass with the regular contour. The mass usually has a characteristic "hard" feel or solid quantity. Additional methods of investigation are used for confirmation of the diagnosis:  uterine sounding  curettage of uterine cavity   MRI Pelvic ultrasonography is the most common method to confirm the uterine myomas presence. The ultrasonographer may suggest location, quantity, size of uterine fibroids, their sructure, presence of destructive changes. Dynamic observation enables to supervise efficiency of the conservative therapy, tumor growing, or, on the contrary, its reduction under the influence of treatment. Uterine fibroids' complications Prolapse of submucous fibroid (cervical protruding myoma) Submucous fibromyoma is accepted by uterus as an ectogenic body. Fibroid descent to the inferior portion of uterus, irritating the isthmus receptors. It results in myometrial contractions, cervical dilation and uterus pushes out fibroid into . Pedunculated tumor is connected with uterus. If pedicle is short, it can result in difficult — oncogenetic inversion due to prolapse of the submucous fibroid. Speculum examination should be performed for confirmation of this diagnosis: cervical protruding myoma is visible. Treatment. Submucous tumor can be easily removed by the incision of long pedicle by clamping the base through the cervix. The pedicle is then ligated. Such removal of fibroid can lead to uterine perforation when the pedicle is short and wide. These patients need . Torsion of uterine fibroid Torsion of uterine fibfoid is a very common in subserous location. Clinically it is characterized by crarfiping pain, signs of peritoneal irritation, fever, urinary frequency and symptoms of rectal pressure. In this situation and infection are common. Surgical treatment. Myomectomy is more commonly done when abdominal myoma location. Myomectomy should be the operation of choice in case of single subserous pedunculated tumor Uterine fibroid' necrosis Necrosis of uterine fibroid results from blood supply disorder of the tumor, occuring due to rapid growing, pregnancy, mechanical accident, and postmenopausal atrophy. It leads to tumor edema and pseudocapsule hemorrhages Clinically it is characterized by cramping pain which enforces during palpation. Signs of peritoneal irritation are found. Fever and leukocytosis accompany severe degeneration. Treatment is surgical removal. Uterine fibroid' suppuration Uterine fibroid's suppuration arises primarily very seldom. Sometimes it is a result of necrosis. Submucous and interstitial uterine fibroids may be suppurated. The serious septic state demands supracervical hysterectomy (subtotal) or total hysterectomy. Pseudocapsule' and uterine fibroid' vessels rupture Pseudocapsule' and uterine fibroid' vessels rupture happens very seldom. It is accompanied by severe pain, signs of intraabdominal hemorrhage (hemorrhagic shock).

Uterine myoma and pregnancy Pregnancy at fibromyoma of uterus comes mainly at subserous and interstitial location of uterine fibroids. Submucous fibroids manifest with pregnancy progressing. Diagnosis of pregnancy in such patients represents appreciable difficulties. During the pregnancy there is a threat of its interrupting as the result of fibroid blood supply disorder (its necrosis, pseudocapsule hemorrhage). The function of urinary bladder and rectum is broken. Fetal position is frequently incorrect — oblique or transversal one. Breach presentation is common if the myoma does not let the fetal head get into pelvic inlet. Preterm rupture of amniotic fluid, primary and secondary dystocia of labor are common. Cesarean section should be pcrfoimed if the nodes are placed behind the course of the genital canal and block the plane of pelvic inlet. Vaginal delivery is recommended in all other cases of labor. Postpartum hemorrhage happens in the third period of labor. Uterine fibroid should undergo involution until their complete regress in women with high-grade lactation during the further duration of puerperium.

TREATMENT OF UTERINE MYOMA Treatment of fibromyoma should be operative and conservative. Indications to operative treatment are: myomatous uterus larger than 12-week of pregnancy, acceleretion of tumor growing, presence of such symptoms as pam, bleeding, secondary anemia; myoma's complications; suspicion on malignant degeneration and combining with and endometrial hyperplasia, sterility. Operative treatment is performed in case when the patients have contraindication to hormonal treatment. These contraindications are: thromboembolism and thrombophlebitis, varicose phlebectasia, hypertension, operation concerning malignant tumors m the past, no effect from hormones. Surgical interventions are divided into radical and conservative — plastic ones. Radical operations are in uterine removal — total hysterectomy or supracervical hysterectomy. Hysterectomy should be performed in 45-year-old women and older during tumor growing in , presence of cervical and endometrial pathological changes (dysplasia, erosion, polyps, scars), combination of fibromyoma with precanserous lesions of uterine cervix and uterus, endometriosis, cervical and isthmic myoma Supracervical hysterectomy is performed in all other cases Conservative-plastic operations are carried out for reduction or preserving of female menstrual and reproductive functions. Their using is justified in young women for anatomo-functional safety of uterus, fallopian tubes, ovaries and ligaments. In this cases remove only nodes and save the uterus. Embolization of the uterine arteries is a promising organ-saving little-invasive method of surgical treatment. Embolization of the uterine arteries in most cases cures myoma in one day at minimal discomfort for the patient. Conservative treatment of uterine fibromyoma has been confirmed patho- genetically and is directed on correction of hormonal state, treatment of anemia and metabolic dysorder, inhibition of tumor growing. Indications. Conservative treatment is recommended at any age, in case of myoma duration with poor symptoms or without any symptoms, at presence of contraindications to operative treatment. Conservative therapy includes a diet with the usage of products, which contain A, E, K, C vitamins, such microelements as copper, zincum, iodine, iron, antianemic therapy, vitamin therapy, uterotonic drugs for decreasing of menstrual hemorrhage, lodium drugs should provoke inhibition of estrogenic secretion at ovaries; phytotherapy — 60 ml of potato juice per day. Electrophoresis of 1-2% solution of potassium iodide is commonly used 40-60 procedures are needed for the treatment course. Symptomatic therapy includes: hemostatics and spasmolytics, nonsteroid anti- inflammatory drugs (in case of pain syndrome), treatment of pathological conditions, which might promote myoma growth (thyroid gland pathologies, onflammatory process of the genitals) and at metabolism normalization. Hormonal therapy. Hormonal therapy is the basis of medicamental hysteromyoma treatment - it is a correcting hormonal therapy directed at the reduction of both systemic and local dyshormonemia. Oral contraceptives do not lead to myoma reduction, but may decrease menstrual blood loss with a considerable increase of hematocrit and other hemogram indices; they may be used for hemostasis. Hestagens have been used in uterine fibromyoma because of its antiestrogenic effect. Duphaston, Utrogestane for 10-12 days in luteal phase of a reproductive cycle for at least 3 months are prescribed. Progestagens are applied in the complex of medicamental leiomyoma treatment, which is accompanied by hyperplastic processes of the endometrium with the purpose of local hyperestrogenism reduction. The doctor uses the preparations, doses, and regimens, which provide the stromal suppression of the endometrium (dydrogesterone 20-30 mg from the 5th to the 25th day of the menstrual cycle, norethisterone 10 mg from the 5th to the 25th day of the menstrual cycle, linestrol 20 mg from the 5th to the 25th day of the menstrual cycle, etc.). The treatment with gonadoliberin agonists effectively reduces the size of the nodes and uterus, but cannot be used longer than 6 months in connection with the development of medicamental menopause syndrome at durable use. Pharmacologic removal of the ovarian estrogen source can be achieved by suppresion of the hypothalamic-pituitary ovarian axis by the use of gonadotropin- releasing (GnRH) agonists. Buzerelinum, gozerelinum and gestrinol belong to the essentially new medicines that are a gonadotropin-releasing luteal hormone agonists. Buzerelinum in a dose of 200 mg is administrated subcutane-ously for the first 14 days of reproductive cycle, then endonasal prescription in the dose of 400 mkg per day for 6 months. Zoladex-Depo is applied subcutaneous in a dose of 3,6 mg once a month for at least 6 months. This treatment is commonly used for 3 to 6 months before the planned hysterectomy, but it can also be used as a temporizing medical therapy until the natural menopause comes. GnRH agonists can not only result in reduction of uterine size, but also lead to a technically easier surgery with significantly diminished blood loss. Androgens could be applied at uterine myoma in the period of perimeno-pause. Its effect can be achieved by pituitary gland suppression. Androgens can result in reduction of uterine size, endomenal atrophy, ovaries follicular depressing. Methylandrostendiolum is prescribed 50 mg per day during 15 days in the follicular phase of reproductive cycle for 3 to 4 months. Methyltestosterone is administrated in 2 pills under the tongue three times per day during 20 days with 10- day time-out for at least 3 months. ENDOMETRIOSIS Endometriosis involves deposits of endometrium outside the uterine cavity. Its manifestations are very variable and often bear no relation to the extent of the disease. Etiology and pathogenesis. Endometriosis is an estrogen-dependent disease. The origin of endometriosis foci and the reasons for its dissemination have not been completely studied yet. There are a couple of theories of endometriosis pathogenesis. I. The transport theory is based on lymphogenous, hematogenous, and iatrogenic (at surgical interventions, hysteroscopy, hysterosalpingography) dissemination. II. The implantation theory is a theory of retrograde menstruation, which explains the retrograde ingress of the endometrium cells into the abdominal cavity. The reasons for pathological implantation: - hypoluteinism (progesterone synthesis reduction); normally in the peritoneal fluid there is a high (higher than in the blood) sex steroid concentration, which inactivates the endometrioid cells; - protracted menstruation, cervical canal constriction, and a wide uterotubal lumen lead to an increase in the amount of retrograde menstrual blood; - immune dyscrasia; - burdened heredity. III. The theory of celomic metaplasia (degeneration) of the peritoneal mesothelium in response to the irritant action of the endometrial tissue. VI. The embryonal theory explains the dysembrioplastic genesis of endometriosis from the remains of the Mullerian ducts and primordial kidney. Histogenesis:  Retrograde spill of menstrual debris through the tubes. Retrograde menstruation takes place in most women, but it is unclear why some women should develop endometriosis while others are unaffected.  Metaplasia of embryonic cells. These are derived from the primitive coelom and may remain in and around the pelvis and differentiate into Mullerian duct tissue.  Emboli of endometrial tissue may travel by lymphatics or blood vessels and become established in various sites. The first of these theories is most favoured. Some authors regard endometriosis as an autoimmune disease, in which immune protection disturbance leads to an increase of macrophage content in the peritoneal fluid, activation of acid phosphatase and neutral protease. In women with endometriosis in the peritoneal fluid there is observed disbalance of the cells providing peritoneum resistance. Thus, there lowers the level of natural killers and rises the content of IL-1, IL-6, IL-8. The factor, which significantly promotes endometrium cell retention, is mesothelium integrity damage, since the intact peritoneum is a barrier for invasion. The hormonal factor, as a reason for sterility in endometriosis, undoubtedly plays one of the leading roles, however, the literature data on sex hormone secretion are ambiguous and mainly testify to their increase except for progesterone, whose content reduces. It has been found that hormones do not influence the cells of the hormone- dependent organs directly, but mediately by means of growth factors, gene expression, specific proteins, and . The balance of molecular-genetic indices of the proliferation and apoptosis processes in the endometriosis foci is disturbed. The study of apoptosis processes in endometriosis is a new tendency in the treatment of this pathology. Endometriosis classification: 1. Internal (70-90 %) - of the uterine body, isthmus, and the interstitial parts of the uterine tubes. If the uterine walls thicken, internal endometriosis is called . 2. External (10-30%): - peritoneal (of the ovaries, uterine tubes, small pelvis peritoneum); - extraperitoneal (of the uterine neck, vagina, external genitals, retrocer-vical). The most widespread external endometriosis classification was worked out by A. Acosta and co-authors (1973). According to this classification, there are differentiated such stages of external endometriosis: • Minor endometriosis: - isolated heterotopies on the pelvic peritoneum; - isolated heterotopies on the ovaries without adhesions and scars. • Moderate endometriosis: - heterotopies on the surface of one or both ovaries with small cyst formation; - latent periovarial or peritubal adhesion abscess; - heterotopies located on the peritoneum of the extrauterine space with a scarring process and uterus displacement without the large intestine being affected by the pathological process.  Severe endometriosis: - endometriosis of one or both ovaries with formation of cysts larger than 2 cm; - affection with an evident periovarial and/or peritubal scarring process; - uterine tube affection with deformation, scarring, blockade; - peritoneum affection with extrauterine space obliteration; - affection of the sacrouterine ligaments and peritoneum of the extrauterine space; - affection of the urinary tracts and/or bowels.

According to the pathological process localization there are singled out: I. Genital endometriosis. 1. Internal endometriosis. 1.1. Endometriosis of the uterine body of the I, II, III (adenomyosis) stages depending on the depth of myometrium affection: glandular, cystic, fibrous forms (focal, nodal, and diffuse forms). 1.2. Endometriosis of the cervical canal. 1.3. Endometriosis of the intramural part of the uterine tubes. 2. External endometriosis. 2.1. Peritoneal endometriosis: - endometriosis of the ovaries (infiltrative and tumor forms); - endometriosis of the uterine tubes; - endometriosis of the pelvic peritoneum (red, black, and white forms). 2.2. Extraperitoneal endometriosis: - endometriosis of the vaginal part of the uterine neck; - endometriosis of the vagina, ; - retrocervical endometriosis; - endometriosis of the uterine ligaments; - endometriosis of the parametral, paravesical, and paracolpal fat with and without invasion to the urinary bladder, rectum. 3. Externointernal endometriosis. 4. Combined forms of genital endometriosis (genital endometriosis in combination with some other genital or extragenital pathology). II. Extragenital endometriosis (endometriosis of the , urinary organs, skin, navel, postoperative wounds, lungs, pleura, etc.). Pathology The gross appearance shows ectopic deposits which can very in number from a few in one locality to large numbers distributed over the pelvic organs and peritoneum. The commonest sites of these deposits are: The commonest appearance of a typical lesion is that of a round protruding vesicle which shows a succession of colours from blue to black to brown. The variation in colour is due to haemorrhage with subsequent breakdown of the haemoglobin. Ultimately the area of haemorrhage heals by the formation of scar tissue. The result is a puckered area on the peritoneum. Commonly however the haemorrhage results in adhesion to surrounding structures. These adhesions are more apt to form between fixed structures such as the broad ligament, ovary, sigmoid colon or the posterior surfaces of the vagina and cervix. The ectopic deposits of endometrial tissue vary in size from pin-point to 5 mm or more. It is these larger deposits which tend to rupture leading to adhesions. These adhesions over the ovary can lead to the formation of quite large haemorrhagic cysts due to continued bleeding from deposits, the blood being unable to escape. Investigation has shown that many lesions do not have a 'typical' appearance. The following is a list of other appearances which have been described. White, slightly raised opacities due to retro-peritoneal deposits. Red flame-like or vascular swellings, more common in the broad ligament or utero-sacral ligament. Small excrescences like the surface of normal endometrium. Adhesions under the ovary or between the ovary and the ovarian fossa peritoneum. Cafe-au-lait patches often in the Pouch of Douglas, broad ligament or peritoneal surface of the bladder. Peritoneal defects on utero-sacral ligament or broad ligament. Areas of petechiae or hypervascularisation usually on the bladder and the broad ligament. Secondary pathology This is due to the adhesions between the endometriotic deposits and adjacent organs. In long-standing cases the pelvic cavity is obliterated by these adhesions. Retroversion of the uterus can be produced.

Clinical findings Quite often deposits are found incidentally in women who have no symptoms of endometriosis and are undergoing laparoscopy or for some other condition. In addition, as indicated in the section on pathology, many peritoneal changes now known to be due to endometriosis were undiagnosed in the past. The prevalence of endometriosis peaks between the ages of 30 and 45 years. Since ectopic endometrium is stimulated by the same ovarian steroid hormones as the endometrium lining the uterine cavity, endometriosis is almost never found outside the reproductive years. Symptomatology A. Pain affects more than 80% of women with endometriotic deposits. The pain tends to begin premenstrually reaching a peak during menstruation and subsiding slowly. The character of pain may vary as does its apparent origin. It may be generalised throughout the abdomen and pelvis like the pain of severe dysmenorrhoea. Alternatively, pain may be localised to a particular site within the pelvis. Deep affects around 40% of women with endometriosis. B. Menstrual disturbance. Menstrual disturbance affects around 20% of women with endometriosis. It may take the form of premenstrual 'spotting', menorrhagia or infrequent periods. Lesions in the wall of the bladder may result in 'menstrual haematuria'. C. Infertility. Endometriosis is found more commonly in women undergoing investigation for infertility than in the 'normal' population. It is not clear which condition arises first. Approximately 30% of patients with endometriosis complain of infertility. When endometriosis is extensive, and both fallopian tubes are occluded, the mechanism by which endometriosis prevents conception is obvious. However, milder forms of endometriosis are also associated with subfertility, and here the pathophysiology is less clear. The most likely mechanism appears to be that immunological factors within the peritoneal cavity inhibit normal gamete function, thus reducing fertilisation rates. DIAGNOSTICS Physical examination Endometriosis cannot be diagnosed by physical examination alone. However, enlargement of the ovaries, fixed retroversion of the uterus and tender nodules within the pelvis may each raise the suspicion of the disease. Endometriosis should always be considered when patients have symptoms referable to the pelvic cavity. In order to diagnose genital endometriosis (GE) one uses the following instrument investigation methods: 1) roentgenological (GSG); 2) US (transabdominal and transvaginal); 3) endoscopic (, hysteroscopy, laparoscopy); 4) urological (, chromocystoscopy); 5) nuclear magnetic resonance (NMR, or MRI) tomography; 6) cytological; 7) histological study of biopsy material. In order to diagnose GE, the degree and character of vascular disorders one resorts to dopplerometric investigations of the blood flow in the arteries of the organs of the small pelvis. Among roentgenological techniques of GE diagnostics there is used hysterosalpingography performed on the 2nd-3rd day after menstruation or after diagnostic curettage. In this case preference is given to the use of water solutions of radiographic contrast media. The most typical roentgen sign in internal endometriosis is the presence of edge shadows of different length. Their form depends on the localization of the endometrioid foci. Hysteroscopy shows endometrioid foci in the form of dark red pin holes against the background of pink-pale hue of the uterine cavity mucosa; at multiple localization of endometriosis foci the endoscopic picture reminds the honeycomb structure. The nodular form of adenomyosis is characterized by enlargement and deformation of the uterine cavity due to bulging of its affected walls, on which there are localized pathological formations with a yellow or yellow-pale hue without clear limits, sometimes with the presence of superficial endometrioid foci. Laparoscopy Laparoscopic examination is the only way of making a positive diagnosis. The lesions can be seen and their number and location estimated. Endometriosis of long standing may be very difficult to diagnose due to obliteration of the pelvic cavity by adhesions. Histological confirmation must be obtained if feasible. Imaging techniques , computerised tomography and magnetic resonance imaging may suggest the presence of endometriosis (e.g. by the demonstration of a particular type of ) but are by themselves insufficiently reliable to make the diagnosis. The biochemical markers of endometriosis are not known at the moment. According to statistics, the level of CA-125 - a substance, which is synthesized by celomic epithelium derivatives and is an marker - rises in medium- severe and severe endometriosis and remains normal in minimal and minor process. If endometriosis is suspected, one is recommended to detect the oncomarker twice: during menstruation and in the follicular phase of the menstrual cycle. If the ratio of the CA-125 level indices during menstruation to the index during the follicular phase exceeds 1.5, endometriosis is possible. Reduction of the CA-125 level may testify to treatment efficiency, and its further lowering - to a relapse. Due to the mixture of symptoms and the variation in appearance of the pelvic structures, conditions such as pelvic inflammatory disease and tumours of the ovary and bowel must be considered and eliminated. TREATMENT The therapeutic approaches include observation, palliative, hormonal, surgical, and combination therapy. Taking into account that endometriosis is a disease of the whole organism, and not of a single organ, the therapy must be complex and include: 1) hormonal treatment; 2) immunocorrectors; 3) prostaglandin inhibitors; 4) antioxidants (a combination of vitamins A, E, С, Р), a complex of vitamins- antioxidants (selmevit, complevit); 5) physical therapy. Presently, there is a certain consensus in the treatment of GE in both sterile patients and women without reproductive intentions. Management of chronic pelvic pains and endometriosis: - monophase COCs + nonsteroid anti-inflammatory therapy; - if ineffective – surgical treatment. Any treatment must be aimed at treating symptoms. Since ovarian hormones are responsible for growth and activity in endometrium many medical therapies are designed to reduce ovarian steroid production or oppose their action. 1. Combined contraceptive pill (COCs) The combined oral contraceptive pill also induces decidualisation of ectopic endometrium. It may be given continuously for up to 3 months. 1. (progestins) – Norcolut, primolut-Nor, Provera, Depo- Provera, Duphaston, Utrogestane Progestogens in a relatively high dose induce decidualisation, and sometimes resorption of ectopic endometrium. Side effects include weight gain, bloating and irregular . 3. Gonadotropin antagonists - , Danazol is a antigonadotropin, steroid hormone closely related to testosterone, which inhibits pituitary gonadotrophins, is anti-estrogenic, anti-progestational, slightly androgenic and anabolic. The dose of danazol given can be titrated to the patient's symptoms up to a maximum of 800 mg daily. If danazol can be tolerated, symptoms and objective signs of disease can be alleviated in the majority of patients. However, androgenic side effects including amenorrhoea, weight gain, acne, hirsutism and deepening of the voice may limit acceptability of the drug. Gestrinone is a derivative of 19-nortestosterone. It has slight androgenic activity and is markedly anti-oestrogenic and anti-progestogenic. It interacts with the pituitary steroid receptors and decreases gonadotrophic secretion resulting in diminished follicular growth and anovulation. A bi-weekly oral dose of 2.5 to 5.0mg for 6 months induces amenorrhoea, disappearance of pain and regression of the endometrial deposits. Side effects include weight gain, acne, seborrhoea and mild hirsutism. 5. Gonadotrophic releasing hormone analogues (GnRH agonists) – Diphereline, Zoladex, GnRH analogues are administered by depot injection or nasal spray. Their mode of action is shown above. Although these drugs are generally effective in treating symptoms, menopausal side effects, in particular bone loss, may preclude long term use. In the future, use of 'add back' regimens which include small supplementary doses of oestrogen may prove to be effective in treating the symptoms of endometriosis without the complications of total oestrogen deprivation. 6. Antiestrogens – Fareston, Tamoxifen 10 mg twice a day during 6 month. 7. Prostaglandin inhibitors – NSAI 8. Enzymotherapy. 9. Immunotherapy – immunomodulators. It is expedient to resort to and hardware-controlled physiotherapy. Physiotherapeutic procedures aim at the activation of metabolic processes, reduction of inflammatory phenomena, prevention of hereditary process development, restoration of regulating bonds between the control centers and target organs of the reproductive system (laser therapy, pulse sequence ultrasound, constant and alternating magnetic fields, iodine and zinc electrophoresis, radon baths in hyperestrogenism, microclyster and vaginal irrigations). In sterile women after hormonal treatment of endometriosis there is applied the therapy aimed at hormonal disbalance correction and ovulation stimulation. If the conservative treatment of endometriosis is ineffective, surgical treatment is administered (laparoscopy or laparotomy). Conclusion As with medical therapies for other conditions, the optimum treatment is dictated by the side effect profile which is most acceptable to the patient. None of the drug treatments described will prevent recurrence of endometriosis once therapy has been stopped, although there may be a period of some months between stopping treatment and the re-emergence of symptoms. No medical treatment has been shown to improve subsequent fertility. Notwithstanding, none of the above, with the exception of the combined pill, is a proper contraceptive agent and patients should be advised to use barrier contraception to avoid the potential teratogenic effects of drugs such as danazol if they are at risk of becoming pregnant. Surgical treatment The surgical treatment of GE has always been and remains the only technique, which allows removing the morphological substrate of endometriosis mechanically or eliminating it by means of laser, electrical, or thermal influence. Medicamental therapy in the pre- and postoperative periods makes it possible to optimize the results of surgical treatment. As for the choice of intervention volume, it is believed that even at extensive endometriosis in women concerned with reproductive function restoration one should follow the principles of reconstructive conservative surgery and resort to radical operations only in the cases, when all other possibilities of both surgical and medicamental treatment have been exhausted. The technique and volume of surgical intervention are chosen individually. The purpose of laparoscopic treatment is the removal of all visible foci and restoration of normal anatomical correlations in the pelvic cavity. The treatment of generalized and combined forms of endometriosis with dysfunction or endometriosis affection of the adjacent organs (ureters, bowels, urinary bladder). If endometrioid cysts are large, endometriosis is combined with other gynecological disorders, their adequate treatment requires application of surgical methods, and at the same time there are no conditions to perform the operation in full volume laparoscopically - the method of choice is laparotomy. There are differentiated absolute and relative indications to the surgical treatment of patients with internal endometriosis. Absolute indications include combination of GE with diseases of the internals, which require surgical intervention (hyperplastic processes of the ovaries and/or endometrial precancer, uterine leiomioma, with rapid growth; severe dysplasia of the uterine neck, atypical endometrial hyperplasia, endometrioid ovarian cysts larger than 5 cm, the ovaries function invariably, pathological involvement of other organs and systems with their dysfunction, suppurative involvement of the affected by endometriosis, adhesive process with involvement of the ampullar parts of the uterine tubes, which accompanies endometriosis — the main reason for sterility; endometriosis of the navel, endometriosis of the postoperative scar, combination of endometriosis with some anomalies of the genital organs, a somatic pathology excluding the possibility of continuous hormonal therapy). Among relative indications there are differentiated the presence of asymptomatic uterine leiomyoma in combination with atypical endometrial hyperplasia at the age younger than 40, the 3rd degree of GE spread, ineffective medicamental treatment, which has been uninterruptedly conducted during 6 months. Where infertility is not a problem radical surgery to remove both ovaries is said to be a lasting cure for endometriosis, since it removes the estrogenic stimulus to endometrial growth. In many cases the patient wishes relief from pain but also desires to retain the possibility of future pregnancy. In these circumstances only conservative surgery can be employed. The intentions in conservative surgery are:  To ablate as many endometrial deposits in the pelvic cavity as possible.  To restructure the pelvic anatomy by destroying adhesions which interfere with ovarian and tubal function.  To destroy endometrial deposits in the ovaries.  To deal with sensory nerve pathways. In view of the many vital structures such as the bladder, rectum, colon and ureters in close proximity to each other, conventional open surgery is not always feasible. Laser surgery under laparoscopy, with its almost microscopic accuracy, may be employed. Endometrial deposits and adhesions can be vaporised easily without damaging tissue outside a radius of a fraction of a millimetre from the target. Similarly the laser destruction of ovarian lesions can be carried out without destroying any of the functional tissue. The question of dealing with sensory nerve pathways is difficult to answer. Severe pain is a feature of a number of gynaecological conditions, especially those related to . Elsewhere in this book operative techniques are described which involve interfering with sensory conductivity centrally, i.e. at the spinal cord level. Recently, a local operative procedure, paracervical uterine denervation, has been recommended. This consists of vaporising the utero-sacral ligaments by laser at their attachment to the posterior aspect of the cervix where the sensory fibres emerge from the uterus. Two difficulties are associated with this procedure. First, the ureters must be avoided and, secondly, veins lying lateral to the ligaments must not be injured. Unfortunately severe pain is often associated with severe endometriosis and adhesions may make the operation very difficult. In internal 3rd degree endometriosis (adenomyosis), when the endometrioid tissue extends through the full thickness of the myometrium to the serosa, hormonal treatment is ineffectual, therefore in such cases surgical treatment is indicated - partial hysterectomy, and in endometriosis of the isthmical-cervical part - complete hysterectomy. At endometriosis of the uterine neck in patients of the childbearing age, if there are solitary superficial foci, it is expedient to remove them by means of cryo- or laser destruction followed by colposcopic monitoring and hormonal treatment. The method of choice in the treatment of sterility must be endoscopic interference. The advantages of this method are being minimally traumatic and with the least blood loss, safety if one follows the rules of surgical technique and the surgeon has sufficient experience, and also shortening of the postoperative period. The most important advantage of this method is the visual control, which allows complete elimination of endometriosis foci. Electrocoagulation of solitary endometriosis foci is carried out with the use of mono- and bipolar electrodes. Small (up to 2 cm) endometriomas are to be incised, the doctor evacuates the contents, thoroughly enucleates the membrane of the tumor-like formation and coagulates its bed. Sometimes in order to remove endometrioma one resects the ovary. Endometrioid implants are often located close to the vital organs, which creates certain difficulties for the coagulation of such implants because of the hazard of affecting adjacent organs and anatomical structures. According to indications laparoscopic coagulation is possible and, in some cases, transaction of the sacrouterine ligaments, which allows decreasing the pain syndrome. Treatment efficiency с r i t e r i a: no relapses of the disease, reproductive function recovery, positive dynamics of life quality. Reports in the literature record complete relief from pain in 50% of patients followed for more than a year and another 41% obtained moderate relief. Ovarian Tumors and Tumor-Like Ovarian tumors rank second in the structure of female genitalia neoplasms, according to different authors, their incidence in recent years makes 6–10 % cases. The incidence of benign ovarian tumors is 70–80 %, and one third of them are tumor-like processes. A peculiarity of ovarian neoplasms is their high propensity for malignization, and the prophylactic and diagnostic work of gynecologists on timely detection of tumors of this localization remains subpar. The histological type of an is one of prognostic factors of the disease course. The histological classification of ovarian tumors was approved by the WHO in 1973 and has remained the fundamental one until now. Taking into account its complexity, one is recommended to use a simplified classification for practical application. Histological Classification of Ovarian Tumors І. Epithelial tumors. А. Serous, mucinous, endometrioid, mesonefroid, and mixed: а) benign: cystadenoma, adenofibroma, superficial papilloma; b) related: intermediate forms of cystic adenomas and adenofibromas; c) malignant: , , papillary carcinoma. B. Brenner’s tumor: а) benign; b) related; c) malignant. ІІ. Sex cord-stromal tumors. А. Granulosa theca cell tumors: granulosa cell tumors, tumors of the - group, generalized tumors. B. Androblastomas, Sertoli-Leydig cell tumors (differentiated, intermediate, low-diferentiated). ІІІ. Lipid cell tumors. ІV. Germinogenous tumors. А. . B. Teratoid tumors (homologous, heterologous). V. Tumor-like neoplasms and pretumor processes: of pregnancy, hyperthecosis, follicular cysts, corpus luteum cyst, endometriosis, inflammatory cysts, paraovarian cyst. So far, there exist diverse terms denoting ovarian neoplasms: cyst, cystoma, tumor, , etc. WHO recommends using the following terms: in true tumors – ovarian tumor; in false tumors (retention, inflammatory) – ovarian tumor-like processes. Ovarian epithelial neoplasms consist of one or a couple of types of epithelium in different combinations, mainly of the germinal epithelium. The incidence of epithelial neoplasms is the highest among ovarian tumors (60–70 %). Serous benign ovarian tumors have the highest incidence. Macroscopically serous cystadenoma is a round tumor with thin walls, uni- or bicameral, of dense elastic consistency, filled with serous fluid. There are differentiated two forms of serous cystadenoma – smooth-walled and papillary. On the internal surface of the papillary cystadenoma there are neoplasms reminding cauliflower. Such tumors undergo malignization in 25–50 % cases, which should be always taken into account when the treatment method is being chosen. Mucinous benign ovarian tumors rank second according to their incidence among all ovarian tumors. In most cases these neoplasms are large, have a multicameral structure, are filled with jelly-like contents. Unlike serous tumors they secrete mucin – mucus in the form of granules. Malignization of mucinous tumors is observed in 5–17 % cases. Endometrioid ovarian tumors develop from germinal cysts and inclusions localized in the ovaries, or from endometrial implants encased in the ovarian tissue. The tumor consists of epithelium and endometrial stroma. Benign endometrioid tumors are observed rather rarely. Related ovarian cystadenomas and adenofibromas are a transitional blastomogenesis stage and are referred to the group of potentially low-grade tumors; there is no evident invasion of the adjacent stroma. Such a structure of tumors is diagnosed with the help of histological study, however, usually cystadenoma forms have papillary excresences onto the internal or external surface of the sac. The structure of related mucinous tumors is practically similar to that of benign neoplasms; the borderline structure of tumors is diagnosed with the help of histological study. Brenner’s tumor is referred to the group of epithelial tumors, whose incidence is 0.6–2.6 %. The tumor has a mixed structure – a fibroma with epithelial inclusions filled with mucus, mucin, and colloid. Sex cord-stromal tumors are primarily hormone-active, because they consist of ovarian cellular elements, which produce female or male hormones. Granulosa cell tumors consist of granulosa cells, which produce . Their incidence makes 2–3 % in the structure of all ovarian tumors. About 30 % granulosa cell tumors have no hormone activity, and in 10 % cases their malignization is observed. Theca cell tumor (thecoma) is an estrogen-producing ovarian tumor; malignant forms are observed in 4 % cases. Incision of the tumor shows a characteristic appearance – the yellow or orange tissue with whitish streaks. is observed in 2–7 % cases of all ovarian tumors. This stromal tumor consists of spindle-shaped cells producing a large amount of collagen. The tumor has no hormonal activity, therefore it is often called a non-functioning tumor. Its shape is round, the consistency is dense, woody, or rocky when calcified. Androblastoma develops from Sertoli-Leydig cells, is referred to masculinising tumors. High-differentiated forms of the tumor are always benign, less homologous ones tend to be malignant. Lipid cell tumors are observed very rarely, consist of cells reminding the structure of the adrenal cortex, and also lutein and Leydig cells. This group of tumors includes hypernephroma, luteoma, masculinoblastoma. In 50 % cases they possess androgenic activity. Germinogenous tumors arise from the primordial germ cells of the embryonic gonads and their derivatives. Dysgerminoma is observed with the frequency of 1.5 %, consists of cells morphologically similar to the primordial germ cells, usually of solid structure. The tumor is malignant; still, is rather favorable. are tumors arising as a result of differentiation of two or three germinal layers. There are singled out mature, immature, and monodermal teratomas. Mature teratomas are more frequently observed in the cystic form (dermoid cyst), less frequently – solid form. Dermoid cyst (a more accurate name – mature cystic ) consists of well-differentiated derivatives of all three germinal layers with ectodermal elements prevalence (hair, fat, teeth, etc.). Mature solid teratoma is an infrequent ovarian tumor, whose appearance depends on the structure of all its mature tissue constituents. Mature teratoma is always benign, its malignization is observed very rarely. Immature teratomas (teratoblastomas) contain immature embryonic structures, are referred to malignant germinogenous tumors относятся. Macroscopy of the tumor may show bones, cartilages, teeth, adipose tissue. Microscopy shows embryonic tissues of different degrees of maturity. Monodermal teratomas are referred to high-differentiated tumors. They include: ovarian stroma (consisting primarily of thyreoid tissue), carcinoid tumor (consisting of argentaffine cells), epidermoid cyst (lined with epidermis), etc. Tumor-like neoplasms and pretumor processes are retentional neoplasms, which arise as a result of fluid accumulation in preformed cavities and enlarge due to fluid quantity increase and not due to cellular elements proliferation. Tumor-like neoplasms are of dyshormonal or inflammatory nature. Follicular cysts are observed in 30 % tumor-like ovarian neoplasms, originate from ovarian follicles. These neoplasms are unicameral, thin-walled, with smooth surface, filled with transparent contents. Corpus luteum cysts morphologically correspond to the yellow body structure, contain granular and theca lutein cells, their walls are thin, the contents are light or hemorrhagic. These cysts form as a result of an inflammatory process or disturbances of the blood or lymph flow in the yellow body, arise in women with biphase cycle. Luteoma of pregnancy develops in pregnancy from lutein cells and depends on the degree of chorionic gonadotropin stimulation. Luteoma of pregnancy sometimes develops in the 3rd trimester, grows to rather large sizes (up to 10–15 cm), and is found during cesarean section (in such a case one should not remove the ovaries, but conduct resection for histological study). After pregnancy termination such neoplasms undergo involution without assistance. Hyperthecosis is bilateral inflammatory excrescence caused by ovarian stroma proliferation. Hyperthecosis is connected with endocrine disorders, is dyshormonal ovariopathy. Inflammatory cysts are a consequence of an inflammatory process of the uterine appendages and are tubo-ovarian tumor-like neoplasms. They resemble a tumor only in outward appearance. Paraovarian cysts are tumor-like neoplasms, which arise in the epoophoron located in the mesosalpinx. Macroscopically, a paraovarian cyst is round, has a smooth surface, thin walls, and dense elastic consistency. Clinical presentation. There are no specific signs of ovarian tumors, as well as tumor-like neoplasms. Even if neoplasms are large, patients may note no significant changes in their condition. However, nonspecific symptoms allow suspecting ovarian tumor development. Patients with ovarian tumors may complain of sensation of heaviness in the lower abdomen, dull pains in the lower abdomen and loin. Acute pain arises in tumor pedicle torsion, tumor sac rupture. Sometimes ovarian tumors are accompanied by constipation, urinary discomfort. If the tumor is rather large, enlargement of abdomen is noted. The same symptom is observed in ascites, which usually develops in malignant ovarian tumor. If the ovarian tumor is hormone-producing, clinical presentations may be connected with the specific action of the hormones produced by the tumor. In granular cell tumor and thecoma there are observed hyperestrogenism signs (hemorrhages, hyperplastic processes in the endometrium). If such a tumor arises at the juvenile age, there appear signs of precocious puberty; if in the period of menopause, menstrual bleedings may recommence. If a patient has a tumor producing androgenic hormones, she has signs of defeminization or androgenization (ischomenia, hirsutism). In about 10 % cases ovarian fibroma is characterized by peculiar clinical presentation called Meigs’ syndrome: ascites with hydrothorax and anemia. Ovarian tumors may combine with other diseases, which develop as a result of ovarian dysfunction (uterine leiomyoma, menstrual dysfunction, endocrine sterility, mastopathy, and mammary tumors), or accompany them (chronic inflammation of the uterine appendages). Diagnostics. Ovarian tumor is diagnosed on the basis of anamnestic data, findings of general, abdomen, and gynecological examination (bimanual vaginal and rectal). Additional investigation methods are usually applied (instrument, hardware- controlled, roentgenological, etc.). One should pay attention to the history of premorbid conditions and risk factors (early and late menarche, early and late menopause, absence of sexual life and , endocrine sterility, condition after ovariotomy or ovariectomy, etc.) and background diseases (ovarian cysts, polycystic ovaries, chronic inflammation of the uterine appendages, uterine tumors, hyperplastic processes in the endometrium, etc.). If there are a lot of risk factors, the patient requires especially thorough and advanced examination. During general examination the doctor pays attention to the character hair distribution, especially excessive, the time of its appearance. When the doctor examines the abdomen, he tries to find the signs of ascites; palpation helps to find tumors not only in the lower parts of the abdomen, but also in the epigastric region. Gynecological examination is conducted when the urinary bladder and bowels are evacuated. This examination may help to find tumors in the region of the uterine appendages and small cavity. One should remember that bimanual examination shows ovarian tumor only if its sizes exceed the physiological parameters of the ovary. Examination helps to find the size of the tumor, its consistency, mobility, character of the surface, location relative to the pelvic organs. Quite frequently, when minimum- size tumors are found, patients are observed during a long time – from a couple of months to a couple of years. It is permissible only due to thorough examination of young women with the help of additional methods (under 25 years of age). One should also pay attention to the condition of the rectouterine space and posterior vaginal wall, where indurations may be palpated frequently testifying to a malignant process. When one finds neoplasms reminding ovarian tumors in the region of small pelvis, the most expedient additional method of investigation is transabdominal and transvaginal echography (ultrasound investigation – US). This very technique is to be used to examine women with overweight, or if there is a commissural process in the small pelvis, because adequate bimanual gynecological examination of such patients is impossible. There are differentiated four echographic types of pelvic tumors: liquid, liquid- solid, solid-liquid, and solid. Ovarian tumors in most cases are referred to the liquid- solid type, which allows giving a diagnosis with a high degree of credibility. In most cases US makes it possible to find the character of the tumor. Echographic picture of smooth-walled tumors (serous cystadenoma, ovarian cysts) is characterized by the presence of space-occupying lesions of different size (one should keep in mind that cysts do not grow larger than 5–8 cm), round form; the sac thickness is 0.1–0.2 cm. the internal surface of the sac is smooth, the contents of the tumor are homogeneous and anechogenic. Papillary serous cystadenoma is defined as a uni- or multicameral neoplasm, with internal septa, the contents are liquid, often with fine-dispersed mixture. On the internal surface of the sac one detects papillary projections in the form of parietal structures of different size, of increased echogenicity. Characteristic signs of are multiple thick septa, unremovable fine-dispersed meal in the cavity of the tumor of high and medium echogenicity. In the echogram, tumors of the genital cord stroma usually have the appearance of unilateral neoplasms with predominantly echo-producing internal structure and echo-poor inclusions. These tumors may also have cystoid structure. Visualization of large tumors with rough surface, irregular shape, especially in case of their bilateral location, makes one suspect a malignant process. If it impossible to give a diagnosis after bimanual vaginal examination and US, one should resort to endoscopic methods of investigation (laparoscopy), which in most cases may not only a diagnostic, but also a therapeutic manipulation (endoscopic removal of an ovarian tumor). A reliable method of diagnostics is computed roentgenological tomography, which is not directly connected with roentgen radiation. Computed tomograms allow finding even small tumors (from 1 cm in diameter). The method of diagnostic aspiration of ovarian tumors using the guidance of US is not recommended because of the risk of violation of sac integrity and dissemination of possible malignant process. If it is impossible to use high-performance methods of diagnostics (US, laparoscopy) in patients with ovarian tumor suspected, laparotomy is to be applied. Treatment. The therapeutic approach depends on the patient’s age, character of the tumor, risk of malignization. In most cases ovarian tumor diagnosis is an indication to surgical intervention. If there is a small mobile tumor, endoscopic operation is possible in young women. Otherwise laparotomy is carried out. When one decides the question of surgical intervention volume, excessive radicalism is inexpedient. If the patient has a tumor-like ovarian neoplasm, it may be removed within the limits of unaffected tissue (resection). When one finds serous ovarian tumors, especially with papillary excrescences, in young women it is expedient to remove one ovary, in patients older than 45 years – both. If the doctor suspects a hormone-producing ovarian tumor, the same measures are undertaken. In young women with small ovarian cysts (follicular, corpus luteum) observation during 3–6 months is possible with constant ultrasound control in different phases of the menstrual cycle, implementation of anti-inflammatory or hormonal treatment. When the therapy is ineffective and tumor-like neoplasm regress is observed, surgical treatment is indicated, mainly endoscopic operations. Paroophoritic cysts are better diagnosed in the course of surgical intervention, because diagnostics before operation is in most cases complicated. Operation in these cases is brought to the enucleation of the cyst from the intraligamentous space with dissection of the broad ligament leaf; the ovary and uterine tube are preserved. Gynecological aspects of diseases of mammoral glands. Investigation and diseases of the mammary glands

The self examination of the mammary glands include: Step 1: Begin by looking at your breasts in the mirror with your shoulders straight and your arms on your hips.  Here's what you should look for:  Breasts that are their usual size, shape, and color  Breasts that are evenly shaped without visible distortion or swelling  If you see any of the following changes, bring them to your doctor's attention:  Dimpling, puckering, or bulging of the skin  A nipple that has changed position or an inverted nipple (pushed inward instead of sticking out)  Redness, soreness, rash, or swelling

Step 2: Now, raise your arms and look for the same changes. Step 3: While you're at the mirror, look for any signs of fluid coming out of one or both nipples (this could be a watery, milky, or yellow fluid or blood). Step 4: Next, feel your breasts while lying down, using your right hand to feel your left breast and then your left hand to feel your right breast. Use a firm, smooth touch with the first few finger pads of your hand, keeping the fingers flat and together. Use a circular motion, about the size of a quarter.  Cover the entire breast from top to bottom, side to side — from your collarbone to the top of your abdomen, and from your armpit to your cleavage.  Follow a pattern to be sure that you cover the whole breast. You can begin at the nipple, moving in larger and larger circles until you reach the outer edge of the breast. You can also move your fingers up and down vertically, in rows, as if you were mowing a lawn. This up-and-down approach seems to work best for most women. Be sure to feel all the tissue from the front to the back of your breasts: for the skin and tissue just beneath, use light pressure; use medium pressure for tissue  in the middle of your breasts; use firm pressure for the deep tissue in the back. When you've reached the deep tissue, you should be able to feel down to your ribca

Step 5: Finally, feel your breasts while you are standing or sitting. Many women find that the easiest way to feel their breasts is when their skin is wet and slippery, so they like to do this step in the shower. Cover your entire breast, using the same hand movements described in step 4 Clinical breast exam: A clinical breast exam is performed by a healthcare professional who is trained to recognize many different types of abnormalities and warning signs. This in-office exam will most likely be completed by your family physician or gynecologist at your annual exam, whereas your breast self-exam is something every woman should do at once at month at home. A visual check of skin and tissue: During a clinical breast exam, your healthcare provider checks your breasts’ appearance. You may be asked to raise your arms over your head, let them hang by your sides, or press your hands against your hips. These postures allow your healthcare provider to look for differences in size or shape between your breasts. The skin covering your breasts is checked for any rash, dimpling, or other abnormal signs. Your nipples may be checked to see if fluid is expressed when lightly squeezed. A manual check for unusual texture or lumps. Sing the pads of the fingers, the doctor inspects entire breast, underarm, and collarbone area for any lumps or abnormalities. It is worth noting that some women have breast tissue that appears to be full of tiny fibrous bumps or ridges throughout the breast tissue, known as fibrocystic breasts. A suspicious lump –the type your physician is checking for– is generally about the size of a pea before anyone can feel it in the breast tissue. The manual exam is done on one side and then the other. Your healthcare provider will also check the lymph nodes near the breast to see if they are enlarged. An assessment of any suspicious area. If a lump is discovered, the doctor should note its size, shape, and texture, ability to move. Benign lumps often feel different from cancerous ones, but any lump found will likely need to be examined with further diagnostic measures. It may be helpful to know that lumps that appear soft, smooth, round, and movable are likely to be either benign tumors or cysts. A lump that is hard and oddly-shaped and feels firmly attached within the breast is more likely to be cancer, but further tests are needed to diagnose the problem. History taking: The number and date of labor and abortions, disorders of the menstrual cycle, onset (acute, chronic), the nature of pain (single, bilateral, continuous or connected with menses), presence of abnormal discharge from the nipple and their nature, loss of weight, anemia, age over 35 years, family history, the presence of cancer pathology in relatives, age of menarche to 12 years, first birth after age 30, menopause after age 55, the results of previous atypia in biopsy material, alcohol abuse (use of more than 100 ml of spirits or wine 200 ml per day), the use of exogenous hormones - continuous use of COCs and HRT - more than 10 years. Diagnostic measures: mammography, ultrasound, MRI, biopsy The American Cancer Society’s current Breast Cancer Screening Guidelines recommend:  Yearly mammograms at age 40 and continuing for as long as a woman is in good health  Breast exam about every 3 years for women in their 20s and 30s and every year for women 40 and over  All women should know how their breasts normally look and feel and should report ANY breast change promptly to their doctor or nurse. Breast self- exam (BSE) is an option for women starting in their 20s. Screening methods according to the Ukrainian guidelines:  Self exam- after 20 years monthly  Clinical exam of a general doctor – all age groups – annualy  Mammography – after 40 years annualy

Ultrasound is used in case of screening for breast cancer in young women (under 40), monitoring breast state in women taking hormones (progestins, COC HRT), the interpretation of mammography data to identify patterns of formations, dynamic monitoring of size of tumors in young women, control examination after biopsy and surgery on the breast. Instrumental methods of investigations: mammography, cytological examination of pathological secretions of the mammary glands, puncture (aspiration) biopsy, galactography - contrast study of the milk ducts, transillumination - translucence the breast. Classification of benign diseases of mammary gland:  Epithelial (adenoma, , adenofibroma, papilloma of the milk ducts);  Nonepithelial (fibroma, , chondroma, osteoma, angioma);  Cystic (lactic cysts, mastopathy).  Adenoma: glandular tumor size 1-3 cm in diameter, lobular, painless, mobile, has a capsule, located on the periphery of the gland easily determined by palpation. Consistency is close to the consistency of the breast.  Fibroadenoma: tumor composed of glandular and large amounts , a capsule. Fibroadenoma makes up the bulk of benign tumors, appear mostly in girls and young women. Consistency is denser than in adenomas. The tumor is clearly separated from the surrounding tissue, mobile, painless. Size varies, usually 1-2 cm in diameter.  Fibroma - connective tissue tumor of hard consistency, rounded shape.  Lipoma - a tumor of adipose tissue.  Chondroma and osteoma are characterized by high firmness and special histological structure.  Angiomas. Angiomas localized deep in the gland have a practical significance. Grow slowly.  Mastopathy. benign breast tumors caused by proliferative changes in gland tissues that develop during the menstrual function. Mastopathy often develops in the age of 30-50 years. Classification. Based on the clinical features distinguish the following forms mastopaty: . Nodal . 2. Diffuse. Papillomas of milk ducts(Mintz disease) represent single or multiple papillary proliferation of the epithelium, often localized in the region of the sinus ducts have a soft consistency and small size (0.5-0.6 cm). Grow slowly for a long time do not manifest themselves. The presence of bleeding from the nipple is often the only symptom that brings the patient to the doctor.  Milk cysts (halaktotsele). Formed in the mammary gland during lactation by type of retention cysts as a result of obstruction of milk ducts after suffering from mastitis or undetectable inflammation of its wall. Scheme 3.Algorithm of management if diagnosed with a tumor in the mammary gland. that are recommended for patients with dyshormonal disorders of the reproductive system in order to prevent the development of breast cancer in Ukraine:  Anti-stress drugs  Vitamins, antioxidants  Immunomodulators, adaptogens  Medications that improve the function of the gastrointestinal tract  Hormone and hormone like medications

Breast diseases Breast diseases are a challenge both for the patient, and for the doctor. Gynaecologists pay more attention to them, because the incidence of benign and malignant breast diseases increases. The breast of an adult woman is a large modi- fied sebaceous gland, which is placed on the super- ficial fascia of the thorax; its weight averages 200– 300 g. The structural elements of the breast are fatty tissue, fibrous septums (septi), and glandular struc- tures. It consists of 12–20 lobes, lactiferous ducts and secretory cells, contained in the alveoles. Each of these lobes has lactiferous ducts which open into the nipple. The breast has rich blood and lymph sys- tems which causes fast metastasing as a result of ma- lignancy. Additional nipples (polythelia) are observed more often than additional mammary glands (poly mastia). The breast tissue is very sensitive to hormonal changes. During puberty breast development is con- nected to action of hormones. The breast is also very sensitive to the level of hormones during the men- strual cycle. Pathological changes can develop in each tissue of the breast: fibro-cystic mastopathy and fibroade- noma — from the connective tissue, lipoma — from the fatty tissue, as well as due to trauma. The duct system of the breast sometimes expand (ductular ectasia, galactocele), can contain papillary tumours or subject to malignant changes. Infectious disease of the breast — mastitis — develops in feeding moth- ers. Breast cancer makes up 25% of all in women and develops mainly after the age of 40–45. BENIGN DISEASES

Mastodynia — pain in the breast, connected to the cyclic hypostasis of the breast due to venous sta- sis and hypostasis of the glands stroma before men- struation; during this time the breasts increase by almost 15%. significant role in the development of this disease is played by heredity and factors of the environment. Clinical picture, diagnosis. Early symptoms of breast cancer can be absent or characterized by the mass of small insensitive mobile masses in the breasts. The growth of a tumour is accompanied by disorder of its mobility, fixation, pink or orange dis- charge from the nipple. Screening of breast cancer is conducted during objective examination of each patient no less than once a year, as well as during self-examination. Mammography (Fig. 71) is performed in patients over the age of 35; till the age of 50 the given in- vestigation should be performed every 1–2 years; after 50 — annually. Treatment is the same, as for premenstrual syn- drome (nonsteroid antiinflammatory means, diuret- ics, pregnin, norcolut, dufastone, urogestan, projestogel, parlodel). Fibrocystic mastopathy (FCM), by definition of the WHO, — fibrocystic disease characterized by a complex of proliferative and regressive changes in the breast with a disorder in the ratio between epithe- lial and connective tissue components. Proliferative processes include hyperplasia, proliferation of the lobes, ducts, connective tissue; regressive — atro- phy, , mass of cysts. FCM is the most widespread disease of the breasts (peak incidence — 45 years old). The risk of breast cancer development with mastopathy with hyperplastic processes increases 2.6 times, in case of atypical changes — 6 times. The term “fibrocystic mastopathy” covers more than 35 various pathological processes. These con- ditions can occur in connection with an increased sensitivity to the action of hormones and conse- quently most are frequently observed in the repro- ductive period or in women who have received oes- trogen replacement therapy in postmenopause. The reason for them also can be inadequate secretion of oestrogen, progesterone or an increase in the prol- actin level. There is no proof of the influence of hormonal contraceptives on the development of be- nign breast diseases. Histological fibrocystic changes go through three stages. The first stage — proliferation of the stroma, especially in the upper quadrant of the gland, result- ing in induration and causing pain in patients. The second stage is adenosis, causing the mass of cysts with various sizes: from microscopic to 1 cm in di- ameter. During the adenosis stage significant pro- liferation of the ducts and alveolar cells develops. The last, third stage of fibrocystic changes is char- acterized by the mass of large cysts. However, with less sensation of pain (except for cases of fast cyst growth). Proliferative changes occur in all the breast structures. In case of the occurrence of atypical cells in the ducts or apocrine tissue, the risk of develop- ing breast cancer increases 5 times. The following forms of FCM are distinguished: diffuse: with the prevalence of the glandular compo- nent (adenosis); with the prevalence of the fibrous component; with the prevalence of the cystic component; mixed; nodular. Etiology and pathogenesis of FCM are not es- tablished. A certain role is played by hormonal dis- orders: hyperoestrogenism, deficiency of progester- one, hyperprolactinemia. Clinical picture. Fibrocystic mastopathy, as a rule, is accompanied by cyclic, bilateral pain (mas- talgia) and breast swelling. The pain connected to the fibrocystic changes, in most cases is diffuse, fre- quently with irradiation into the shoulder. The lo- cated pain can occur due to fast growing cysts. Dur- ing examination of the breasts, their nodularity is found. Large cysts can be felt as filled with liquid. These changes are most expressed before menstru- ation. However, may not have a cyclic character. Up to 15% of women, as a rule, do not complain of pain. However, during palpation of the breasts zones of nodularity are found in them. Diagnosis is based on history data, clinical pic- ture, palpation of the breasts, ultrasound study. The diagnosis is confirmed with data from special in- vestigationes (mammography, thermography, punc- tual biopsy). A biopsy consists of aspiration of cystic masses; this procedure simultaneously is also ther- apeutic. The tissue received by aspiration is subject to cytological investigation. The cystic masses may fully disappear after aspiration; in such cases patients should have a repeated examination in 3–6 months. With the presence of solid masses, histological study is required by aspiration or open biopsy. Mammogra- phy is recommended to women over the age of 35 having a high risk of developing breast cancer. Such breast diseases as sclerosing adenosis, apocrine meta- plasia, ectasia of the ducts, fibroadenoma, fibrosis, mild hyperplasia, mastitis and squamous metapla- sia do not increase the risk of developing breast can- cer. Treatment of patients with fibrocystic mastopa- thy is medicamentous. With this purpose they use oral contraceptives (method of choice), gestagens, danazol, antioestrogen (tamoxifen, nolvadex), anti- preparations (parlodel, bromcriptin), micro- doses of iodine, vitamins. In every second patient who is treated with danazol 12 months after the ter- mination of therapy symptoms repeat. Parlodel, as a rule, has bad tolerance (nausea, vomiting). Such pa- tients are recommended to limit the use methylxan- thin (coffee, tea, chocolate). Fibroadenoma has the second rate among be- nign breast diseases. It develops mostly in young women (21–25 years). It is revealed as a firm, pain- less, freely mobile mass in the breast usually with a diameter of 2–3 cm. In most cases fibroadenoma is single; multiple tumours can be formed in 15–20% of all cases. Fibroadenoma does not undergo chang- es during the menstrual cycle and, as a rule, grows slowly. The diagnosis is established during objective ex- amination and is confirmed by aspiration biopsy with subsequent histological study. Patients can find a tumour independently during breast self- examina- tion. Treatment of patients with fibroadenoma consists of surgical removal of the tumour, though in some cases treatment with danazol or tamoxifen is effec- tive. Lipoma, fatty necrosis. In the fatty tissue of the breasts benign tumours which at times are difficult to distinguish from carcinomas can develop. Lipo- ma, as a rule, is not sensitive during palpation. How- ever, the diffuse character can resemble malignant neoplasm. Secondary changes, characteristic for cancer (change of skin, nipple), in most cases are absent. Fatty necrosis develops extremely seldom due to trauma; its reason is not completely known. The patient complains of firm painful masses in the breast; in some cases retraction of the skin is present. To specify the diagnosis, mammography and biop- sy of the breast are done. Intraductal papilloma is the frequent reason for discharge from the nipple of one breast, including bloody. This tumour is solitary; multiple papillomas (papillomatosis) are possible. With the presence of thick discharge in a combination to a tumour mass in the breasts the differentiation with carcinoma is nec- essary. Mammary duct ectasia occurs in the perimen- opausal period (in women over the age of 55) more often and is characterized by an increase, redden- ing and hardening of the breasts, the presence of grey, greenish, even black, discharge from the nip- ple, having a consistency of toothpaste. Ectasia of the mammary ducts is connected to their chronic inflammation and can be accompanied by local lymph nodes enlargement. The presence of local firm masses and enlarged lymph nodes demand their differentiation with carcinoma. Mammary duct ecta- sia can independently regress. BREAST CANCER

Within the last 10 years the case rate of breast cancer in Ukraine has almost doubled, and in 1996 was 51.2 in 100 thousand of the female population. The basic epidemiological factors influencing the development of breast cancer are submitted below. Etiology. The reason for cancer is unknown.

What do the BI-RADS categories mean?

Category Definition What it means

0 Incomplete - This means the radiologist may have seen a possible Additional imaging abnormality, but it was not clear and you will need evaluation and/or more tests, such as another mammogram with the use comparison to prior of spot compression (applying compression to a mammograms is smaller area when doing the mammogram), needed. magnified views, special mammogram views, or ultrasound. This may also suggest that the radiologist wants to compare your new mammogram with older ones to see if there have been changes in the area over time.

1 Negative There’s no significant abnormality to report. Your breasts look the same (they are symmetrical) with no masses (lumps), distorted structures, or suspicious calcifications. In this case, negative means nothing bad was found.

2 Benign (non- This is also a negative mammogram result (there’s no cancerous) finding sign of cancer), but the radiologist chooses to describe a finding known to be benign, such as benign calcifications, lymph nodes in the breast, or calcified . This ensures that others who look at the mammogram will not misinterpret the benign finding as suspicious. This finding is recorded in your mammogram report to help when comparing to future mammograms.

3 Probably benign The findings in this category have a very high chance finding – Follow-up in (greater than 98%) of being benign (not cancer). The a short time frame is findings are not expected to change over time. But suggested since it’s not proven to be benign, it’s helpful to see if the area in question does change over time.

You will likely need follow-up with repeat imaging in 6 months and regularly after that until the finding is known to be stable (usually at least 2 years). This approach helps avoid unnecessary biopsies, but if the area does change over time, it still allows for early diagnosis.

4 Suspicious Findings do not definitely look like cancer but could abnormality – Biopsy be cancer. The radiologist is concerned enough to should be considered recommend a biopsy. The findings in this category can have a wide range of suspicion levels. For this reason, this category is often divided further:

4A: Finding with a low likelihood of being cancer (more than 2% but no more than 10%)

4B: Finding with an moderate likelihood of being cancer (more than 10% but no more than 50%)

4C: Finding with a high likelihood of being cancer (more than 50% but less than 95%), but not as high as Category 5

5 Highly suggestive of The findings look like cancer and have a high chance malignancy – (at least 95%) of being cancer. Biopsy is very Appropriate action strongly recommended. should be taken

6 Known biopsy-proven This category is only used for findings on a malignancy – mammogram that have already been shown to be Appropriate action cancer by a previous biopsy. Mammograms may be should be taken used in this way to see how well the cancer is responding to treatment.

Preparation and postoperational management of gynecological patients

in urgent and planned operations. Prevention of HIV

Minimally invasive, laparoscopic surgery is, and must always be, considered major surgery. Therefore, it is important to carefully prepare the patient for surgery both psychologically as well as physically. The surgeon must also be prepared by adequate training and practice in the techniques that are necessary to complete the procedure in a safe and efficient manner. Patient preparation begins with the initial decision to perform laparoscopic surgery, and although it is tempting to convert most procedures to a minimally invasive route, the surgeon must consider if the particular pathology should be approached in this manner and is in the best interest of the patient. Just as importantly, the surgeon must honestly evaluate his/her own ability and training.

Surgical interventions in gynecological practice is used only when the conservative methods of treatment have been tried. There is a possibility, that the conservative methods of treatment will be of no use and operation is the only way for the patient's convalescence, and sometimes for saving her life.

Each operation is performed according to the certain indications. Indications are the aggregate of causes which determine the necessity of some surgical intervention. Indications for the operation have to be carefully thought out by a doctor and written down into the case history. It is also necessary to take into account the presence of contra-indications, and only after analysis of all the data one should decide the question about the kind and volume of the operation, taking into account woman's age, presence of children, desire to have children later or on the contrary, contra-indications to pregnancy according to the health state. Operation has not only to remove the cause of the disease, but also to preserve functions of woman's organism — menstrual, sexual and reproductive. "The final aim of any operation is neither the destruction of the sore organ or its removal, but the renewing of the integrity and all its functions", — wrote A.P. Gubarev. Extraordinary important is the correct diagnostics. It is necessary to use all possibilities of the clinic or any other gynecological permanent establishment in order to reveal diseases of genitals or concomitant diseases, that can be contraindications for narcosis or operation itself, to prepare a woman properly for the operation and to avoid annoying unexpectedness during surgical intervention, which sometimes make a surgeon change in advance determined plan and operation volume. Establishing, that there are absolutely no contra-indications to surgical treatment of this patient, a surgeon must choose the most appropriate operation method namely for this patient. Taking into account mentality peculiarities of the patient, that is being prepared for operation, and her protection from traumatizing and stress is the first and foremost task of all the surgical gynecological department staff. Insufficient tact in attitude to patient, indifference can make worse the results of the perfectly executed in technical plan operation. One should remember, that it is very hard for a patient to dispose herself to necessity of surgical intervention psychologically, and even women with strong nervous system are afraid of the operation. That's why the behaviour of a doctor and everybody, who communicates with the patient has to be positive, mood must be optimistic. The patient must not have doubts about the necessity of the operation and its positive effect on her health. Patients, which are hospitalized in planned order, should be accommodated in one ward with those, who recover after operation and on the contrary, they must be isolated from the recently operated patients. Comfortable atmosphere in department, attentive attitude of medical staff, clearness of prescriptions execution create favourable psychological climate and facilitate anxious time of operation waiting. However, it is impossible to take into account only psychotherapeutic effect of a word, it is necessary to conduct medicinal therapy, to provide full value daily and especially night rest. With this aim Novopassit, Valerian drops, Seduxen, Sibazon, Relanium, Trancsen during the day and especially before going to bed are recommended.

OPERATIONAL UNIT The operational unit of gynecological clinic consists of such parts: preoperative room, rooms for patient preparation, material room, operating-room. Preoperative room. In this room the surgeon and his assistants wash the hands, dress in operation clothes, aprons, masks. The operation brigade is ready to dress sterile surgical coats and to begin the operation. Room for preparation of patient for surgery. It is recommended to prepare a patient for operation in room, that is situated near the operating-room. It deprives the patients of negative emotions from the contemplation of the operating room. A material room is set for keeping of operation linen, gauze, cotton wool, instruments. This room is to be isolated from the other rooms. Materials into operating-room are given through special window. Operating-room. There must be at least two operating rooms in surgical gynecological department: "clean" and "purulent", because in practice occurs that aid to both non- infected and infected patients should be given. "Clean" operating room has to be larger, 1-2 surgical tables can be situated in it. In "purulent" operating room not only patients with the fixed diagnosis of purulent process in genital organs are operated, but also those that subject to surgical intervention for emergent indications, because a pus presence can be diagnosed in insufficiently examinated patients during the operation. Then operating-room should be carefully cleaned up and disinfected. Demands on cleaning: • Operating-room, preoperative and material rooms are situated in the zone of sterile health regimen. A zone of sterile regimen needs keeping special demands previous cleaning is dusting the furniture, devices and floor bel beginning of each workday • current cleaning (keeping of cleantiness and order in the operatinj — during the operations the gauze balls, serviettes, instruments, th dentally have fallen down on the floor are cleared away; if a liquid the floor, it must be wiped off immediately • postoperative cleaning — during intervals between operations • finishing cleaning — at the end of the workday: floor washing, humid out walls, window-sills, furniture with disinfectants use. Cleaning i irrespective of the fact, whether there were operations in present day, • general cleaning (disinfection of the operational unit) is made once i After cleaning the operating-room is decontaminated by means of lamp for an hour. Sufficient illumination is of particular importance. It is necessary member, that abdominal operations on internal genital organs are made in the pelvis cavity that's why the illumination by means of operating s lamp is high-effective. During vaginal operations it is better to use the so gynecological lamps, that in addition to daylight allow to focus the I horizontal direction and to enlighten the operative field well. Temperature in operating-room is to be kept within 20-25°C. Sui ventilation or air- conditioning are also necessary. PREPARATION OF THE PATIENT FOR THE SURGERY During preparation of the patient for the planned surgery a careful clinical-laboratory examination including clinical blood test, biochemical blood test, analysis of blood on syphilis and AIDS, determination of blood type and rhesus-factor, analysis of hemostasis system, general urine analysis, investigation of vaginal microflora, smears from external surface of cervix and its canal on atypical cells, ECG, X-ray examination of chest are made. Patient is examined by stomatologist (if it is necessary he sanifies the oral cavity), by therapeutist, anesthetist. For indications the patient is consulted by oculist, neuropathologist, at presence of varicose veins or thrombophlebitis — by vascular surgeon. In case of finding of deviations from norm, correction is made — treatment of anaemia, sanitation of vagina. For patients with malignant tumors concerning the additional methods of examination the X-ray examination of stomach and bowels, proctosigmoidoscopy and chromocystography, in case of infertility, endometriosis, uterine corpus tumors — hysterosalpingography or contrasting sonography are carried out. Inoculation of vaginal discharge for research of microflora and its sensitiveness to antibiotics is made. Especially importuni is the preparation of the aged patients for operation, i with cardiac- vascular pathology, with malignant tumors. Suitable preparatioi prescribing of vasodilators, hypotensive, diuretic agents, cardiac glycoside; cocarboxylase, vitamins allows to prevent complications on the part of cardial vascular system and provides a favorable duration of operative and postoperali v period. On the day before operation a woman does not take shower, she may onl drink a glass of sweet tea, in the morning — no breakfast, she is made a clcansinj, shaving a hair from pubis and external genitals, giving a purgative enema, mak in hygienic douche. Just before the operation the woman empties urinary bladder, permanent catheter for the operation time is inserted. Patients' preparation to vaginal operations, or to operations with acccs through anterior abdominal wall (hysterectomy) has some peculiarities. Previous! for such patients sanitation of vagina to reaching the 1st degree of vaginal eontei is made: syringing of vagina with antiseptic solutions, introduction of tampon with medical emulsions. In operation day vagina is processed with alcohol and sterile tampon is introduced. In case of presence of decubital ulcers on the eervi in patients with uterine prolapse it is necessary to reach their healing. Absenc of infection in vagina considerably decreases the risk of postoperative purulent inflammatory complications development. In the evening before the operation the patient is given sleeping-dnij^ (Phenobarbital0,l- 0,2g, Nitrazepam 0,005-0,01g,Noxyron — 0,25-0,5 g), trail quilizers (Seduxen — 0,0lg, Nozepam — 0,0lg, Elenium — 0,005g). For p;i tients with excitable nervous system tranquilizers are prescribed repeatedly hours before the operation. For 30-40 min before operation premedication is made: Atropine or Methii cin — 0,5- 0,8 mg, narcotic analgesics (Promedol — 20 mg, Fentanyl — 0,1 mj Talamonal — 2 ml), antihistamines (Dimedrol — 0,02 g) are used for this. Preparation of the patient for urgent surgery in majority of cases is greatl limited in time (sometimes up to several minutes). If circumstances allow, emp tying of bowels by means of purgative enema (in case of bleeding enema i contra-indicated) is carried out. Before the operation, that is performed undo endotracheal anesthesia, if the patient had meals not long before, it is neeessar to make gastric lavage or aspiration of gastric contents with aim of Mendclson' syndrome prophylaxy (regurgitation can happen during the operation and acr stomach contents can get into trachea and lungs). If it is necessary hair on ptibi is shaved. If possible, the patient takes a shower. Obligatorily emptying of'urinur bladder or introduction of permanent catheter is carried out. Premedication is made immediately in operating-room, mentioned ubov medications are introduced intravenously.

ANAESTHESIA OF GYNECOLOGICAL OPERATIONS For anesthetizing of great gynecological operations most frequently combined endotracheal anesthesia in combination with neuromuscular relaxant and with artificial pulmonary ventilation (APV) is used. This method allows to maintain sufficient passability of respiratory tract in any patient's position, provides maximum muscles relaxing and absence of pain sensitiveness with minimum amounts of anesthetic, and also they use epidural anesthesia. Such stages in realization of general anesthesia are carried out: • premedication • initial narcosis (intravenous introduction of Thiopental sodium or Hexenal in dose 6-8 mg/kg; in case of blood loss, allergy, bronchial asthma — Ketamine in dose 2 mg/kg) • introduction of depolarizing muscle relaxant (Dithylinum, Lystenon — 2 mg/kg) • mask APV and intubation of trachea • supporting of basic narcosis (APV with Nitrous oxide and Oxygen; intravenous introduction of narcotic analgesics, neuroleptics and tranquilizers; total muscle relaxation; supporting of adequate gaseous exchange and hemodynamics, blood volume correction) • withdrawing of narcosis (canceling of inhalative anesthetics introduction, inhalation of Oxygen; introduction of antidotes for neuromuscular relaxants control for renewing of consciousness, appearing of protective reflexes) • extubation • supporting of stable breathing and blood circulation Anesthetist has to be prepaired for anesthetizing in advance: to check out the readiness of narcosis apparatus, presence of necessary medications and oxygen in oxygen cylinder. On the sterile table there must be all the necessary instruments: laryngoscope, intubation tubes, sterile single-use syringes, mask, medicines. It is necessary to keep an eye on the patient's state permanently and fill in the narcosis card. Small in volume and short-term gynecological interventions also need adequate anesthetizing because of great pain intensity, which appear in the result of irritation of high-sensitive uterine reflexogenic zones. With aim of anesthesia intravenous anaesthetizing by Thiopental sodium, Ketamine or Ketanest, neuroleptanalgesia, ataralgesia are used. Local anesthesia is used only in case of presence of contra-indications concerning the general one: acute inflammatory processes in upper respiratory tracts, full stomach, and also in case of absence of conditions for performing narcosis or patient’s refusal from general anaesthesia. The basic method of local anaesthetizing at uterine curniage, punctures is patacervical anaesthes that is reached by introduction of anesthetic (Lidoeaine 0,25-0,5%) into pin metrial cellular tissue, where nervous plexes arc situated. For paraeervu anaesthesia it is necessary to prepare a Novocaine or other anesthetic, syringe 20 ml capacity and a long needle.

LAPAROSCOPY PATIENT EVALUATION Initial patient evaluation considers the indications and contraindications of laparoscopic surgery. There are no hard and fast rules and even the term “absolute contraindication” must be considered as a guideline, rather than a final decree. ABSOLUTE CONTRAINDICATIONS There are very few absolute contraindications as previously noted. With increased anesthesia ability, even some of these may not be considered absolute. • Severe cardiac disease: (Class IV) these patients may not tolerate the deep Trendelenburg positions necessary during most operative laparoscopy to maintain an adequate pneumoperitoneum that is frequently required for satisfactory vision and instrument movement. • A hemodynamically unstable patient with the need for control of bleeding probably should be approached by laparotomy. However, many surgeons believe that they can rapidly enter an abdomen safely laparoscopically, even in the face of a ruptured ectopic, for example. • Intestinal obstruction with distended bowel is best approached by laparotomy, however, with some of the open laparoscopy techniques, it may be possible to utilize laparoscopy even in these conditions.

RELATIVE CONTRAINDICATIONS • Multiple previous major surgeries must be considered a possible contraindication, depending upon both the entry technique and the skill of the operating surgeon. However, utilization of left upper quadrant insufflation techniques or open laparoscopy may afford safe entry even in the event of multiple previous surgeries. • Morbid obesity may be daunting to the inexperienced laparoscopist, patients as heavy as 120+kg often may be candidates for laparoscopy. • Pregnancy beyond five months gestation must be approached with a great deal of caution as the pelvis is almost completely filled with the enlarged uterus. Some surgeons have used gasless laparoscopy in more advanced pregnancies. However, some studies have shown that the CO2 gas of the pneumoperitoneum does not harm the fetus. • Severe, chronically ill patients may present anesthesia problems, but may still be approached cautiously with laparoscopic surgery. It is important not to compromise the respirations with a pneumoperitoneum that is too large. • The patient should not be compromised by laparoscopic surgery if malignancy is a possibility. If a mass is known to be malignant and the surgeon does not have the skills necessary for complete removal without rupture of the mass, then laparoscopy is not the operation of choice. Some gynecologic oncologists have the skills not only to remove a mass, but also to perform lymph node dissections. In the hands of such surgeons, laparoscopy is acceptable.

HYSTEROSCOPY Hysteroscopy is the inspection of the uterine cavity by with access through the cervix. It allows for the diagnosis of intrauterine pathology and serves as a method for surgical intervention (operative hysteroscopy). Method A hysteroscope is an endoscope that carries optical and light channels or fibers. It is introduced in a sheath that provides an inflow and outflow channel for insufflation of the uterine cavity. In addition, an operative channel may be present to introduce scissors, graspers or biopsy instruments. A hysteroscopic resectoscope is similar to a transurethral resectoscope and allows entry of an electric loop to shave off tisse, for instance to eliminate a fibroid. A contact hysteroscope is a hysteroscope that does not use distention media. Insufflation media The uterine cavity is a potential cavity and needs to be distended to allow for inspection.

Thus during hysteroscopy either fluids or CO2 gas is introduced to expand the cavity. The choice is dependent on the procedure, the patient’s condition, and the physician's preference. Fluids can be used for both diagnostic and operative procedures. However,

CO2 gas does not allow the clearing of blood and endometrial debris during the procedure, which could make the imaging visualization difficult. Gas embolism may also arise as a complication. Since the success of the procedure is totally depending on the quality of the high-resolution video images in front of surgeon's eyes, CO2 gas is not commonly used as the distention medium. Electrolytic solutions include normal saline and lactated Ringer’s solution. Current recommendation is to use the electrolytic fluids in diagnostic cases, and in operative cases in which mechanical, laser, or bipolar energy is used. Since they are conducting electricity, these fluids should not be used with monopolar electrosurgical devices. Non- electrolytic fluids eliminate problems with electrical conductivity, but can increase the risk of hyponatremia. These solutions include glucose, glycine, dextran (Hyskon), mannitol, sorbitol and a mannitol/sorbital mixture (Purisol). Water was once used routinely, however, problems with water intoxication and hemolysis discontinued its use by 1990. Each of these distention fluids is associated with unique physiological changes that should be considered when selecting a distention fluid. Glucose is contraindicated in patients with glucose intolerance. Sorbitol metabolizes to fructose in the liver and is contraindicated if a patient has fructose malabsorption. High-viscous Dextran also has potential complications which can be physiological and mechanical. It may crystallize on instruments and obstruct the valves and channels. Coagulation abnormalities and adult respiratory distress syndrome (ARDS) have been reported. Glycine metabolizes into ammonia and can cross the blood brain barrier, causing agitation, vomiting and coma. Mannitol 5% should be used instead of glycine or sorbitol when using monopolar electrosurgical devices. Mannitol 5% has a diuretic effect and can also cause hypotension and circulatory collapse. The mannitol/sorbitol mixture (Purisol) should be avoided in patients with fructose malabsorption. Procedure Hysteroscopy has been done in the hospital, surigal centers and the office. It is best done when the endometrium is relatively thin, that is after a menstruation. Typically hysteroscopic intervention is done under general endotracheal anesthesia or Monitored Anesthesia Care (MAC), but a short diagnostic procedure can be performed with just a paracervical block using the Lidocaine injection in the upper part of the cervix. The patient is in a lithotomy position. After cervical dilation, the hysteroscope with its sheath is guided into the uterine cavity, the cavity insufflated, and an inspection is performed. If abnormalities are found, an operative hysteroscope with a channel to allow specialized instruments to enter the cavity is used to perform the surgery. Typical procedures include , submucosal fibroid resection, and endometrial polypectomy. Hysteroscopy has also been used to apply the Nd:YAG laser treatment to the inside of the uterus. When fluids are used to distend the cavity, care should be taken to record its use (inflow and outflow) to prevent fluid overload and intoxication of the patient. Indications Hysteroscopy is useful in a number of uterine conditions:

 Asherman's syndrome (i.e. intrauterine adhesions). Hysteroscopic adhesiolysis is the technique of lysing adhesions in the uterus using either microscissors (recommended) or thermal energy modalities. Hysteroscopy can be used in conjunction with laparascopy or other methods to reduce the risk of perforation during the procedure.

. Polypectomy.

 Gynecologic bleeding

 Endometrial ablation (Some newer systems specifically developed for endometrial ablation such as the Novasure do not require hysteroscopy)

 Myomectomy for uterine fibroids.

 Congenital Uterine malformations (also known as Mullerian malformations). Eg.septum,

 Evacuation of retained products of conception in selected cases.  Removal of embedded IUDs. The use of hysteroscopy in is not established as there is concern that cancer cells could be spread into the peritoneal cavity. Hysteroscopy has the benefit of allowing direct visualization of the uterus, thereby avoiding or reducing iatrogenic trauma to delicate reproductive tissue which may result in Asherman's syndrome. Hysteroscopy allows access to the utero-tubal junction for entry into the ; this is useful for tubal occlusion procedures for sterilization and for falloposcopy. Complications A possible problem is uterine perforation when either the hysteroscope itself or one of its operative instruments breaches the wall of the uterus. This can lead to bleeding and damage to other organs. If other organs such as bowel are injured during a perforation, the resulting peritonitis can be fatal. Furthermore, cervical laceration, intrauterine infection (especially in prolonged procedures), electrical and laser injuries, and complications caused by the distention media can be encountered. The use of insufflation media can lead to serious and even fatal complications due to embolism or fluid overload with electrolyte imbalances. The overall complication rate for diagnostic and operative hysteroscopy is 2% with serious complications occurring in less than 1% of cases.

ABDOMINAL GYNECOLOGICAL OPERATIONS For performing of abdominal gynecological operations most frequently lower midline laparotomy and incision by Pfannenshtiel are used. Midline vertical laparotomy Midline vertical laparotomy provides a sufficient access to organs of small pelvis, gives a possibility to have a view of other organs of abdominal cavity by widening the dissection up, one can carry out the revision of all the organs of abdominal cavity and to conduct necessary interventions. That's why this access is used when during operation there are foreseen technical difficulties (in case of peritonitis, internal bleeding, big tumors etc). Technique. Along the middle abdominal line (linea alba) the skin and hypodermic fat is dissected with scalpel from pubis towards umbilicus. An incision size depends on the volume of surgical intervention, in case of tumor removal — from its size. The aponeurosis is dissected. At first a small cut with scalpel is made, then it continues with scissors. The muscles are disconnected. The peritoneum is grasped with two anatomic pincers and is cutted between them with scalpel, then incision is continued up and down with scissors. While continuing the incision towards pubic tubercle, one must be careful for preventing damaging of urinary bladder. To prevent it, only area, that is translucent, under the sight control is dissected. Stitches on abdominal wall layer-by-layer in reverse order are putted.

Laparotomy by Pfannenshtiel Advantages of this kind of incision is absence of cosmetic defect, especially in case of stitching with subcuticular (cosmetic) suture, better healing of the wound, there never happen such complications as eventration because wound layers are dissected in different directions. Skin and subcutaneous fat are cut along the suprapubic fold on distance 2-3 cm from pubic symphysis. In inguinal regions from both sides of incision there pass the superficial epigastric arteries, damaging of which should be avoided, and if they dissect them, it is necessary ligate them immediately. Aponeurosis is cutted slightly with scalpel from both sides from the white line, then the incision is continued with scissors into both sides of wound. The upper edge of aponeurosis in the wound center is clenched with Kocher's forceps and pulling it up they snip it off with scissors from the white-line towards umbilicus, as far as skin dissection allows. Muscles of anterior abdominal wall are not dissected, they are separated in longitudinal direction, as it midline vertical laparotomy. Peritoneum is clenched with two pincers rutted in longitudinal direction at first with scalpel, and then with scissors. Taking into consideration the indisputable advantages of this approach, necessary to note, that in ease of Pfannenshtiel incision appearing of subfasi haematoma, difficult access to organs of small pelvis are observed frequently during the operation some problems such as the necessity of abdominal ca\ revision, big size of the tumor are appeared, it is impossible to extent of I incision. Salpingo- is carried out in case of , purulent proces; in the tube. Adnexa are removed in case of ovarian tumors, because the li belongs to the structure of its surgical pedicle. In case of tuboovarian tumors, the result of inflammatory process ovary and tube adhere between themself. li technically impossible to remove only the tube or only the ovary in this cai Basic indications to oophorectomy are its tumors. Ovarian tumors have the so-called "pedicle". Ovarian and infundibulopi vicum ligaments, part of broad uterine ligament, vessels and nerves, that pass them belong to the structure of anatomic pedicle. Uterine tube also belongs the structure of surgical pedicle of the tumor. In case of complication in the form of tumor pedicle torsion, vessels, th belong to structure of surgical pedicle are pinched, blood supply is disturbe that's why cystoma's capsule swells are infiltrates with blood. Tumor acquires purple colour, enlarges in size. Peculiarity of surgical intervention in case < cystoma's crus torsion is that forceps should be applied beneath the torsion plac to prevent getting into woman's blood toxic products and thromboplasl substances, that have appeared in cystoma after its blood supply disturbance, is forbidden to untwist the pedicle in no circumstances! Stages of the operation: • disinfecting of skin • midline vertical or transverse incision of anterior abdominal wall • introduction of retractor and separation of intestinal bowels • grasping with hand and exteriorization out of uterine tube • application of Kocher's forceps on mesentery of uterine tube and its utcrin end (at adnexectomy forceps are applied on infundibulopclvicum ligameni uterine end of the tube and ovarian ligament, • removal of uterine lube (adnexa), suturing and • peritonization of the slump • lavage of abdominal cavity • report of operating nurse about the presence of all the instruments and serviettes • suturing of abdominal wound • catheterization of urinary bladder EXAMPLE OF THE OPERATIVE NOTES in removal of uterine adnexa (adnexectomy) at ovarian tumor on pedicle Indications for the operation: ovarian cystoma. Anesthetization: endotracheal anesthesia with Nitrous oxide and Oxygen mixture, and neuroleptanalgesia. • Operation passing. The operative field has been processed with 2% iodine spirit solution and by 70% ethyl alcohol, edged by sterile surgical garb. Anterior abdominal is incised layer-by-layer by midline vertical incision. Before cutting the peritoneum, the wound is edged by sterile serviettes and isolated from hypodermic fat. Bowels are separated by serviettes. • During examination of small pelvis ovarian tumor 20x 15 cm in size was found, Tumor is exteriorized into the wound. It was determined, that the tumor takes ib origin from the right ovary, has a long pediclewhich contains ovarian, infundibulo pelvicum ligaments, uterine tube. The uterus and left adnexa are not changed Kocher's forceps are put on the tumor pedicle. The tumor and uterine tube an cutted off. Catgut suture is laid on the stump. Peritonization by round ligament oi uterus is made. • Lavage of abdominal cavity is conducted. Calculation of serviettes anc instruments is made. All are presented. • The incisional wound of anterior abdominal wall is sutured layer-by-layer peritoneum, muscles, aponeurosis — by continuous vicryl suture, skin anc hypodermic fat — by interrupted silk suture. • Asepsis bandage is dressed. • There was no bleeding during the operation. • Operation duration — 40 min. • Name of the performed operation: right-side adnexectomy. • Macropreparation: removed tumor is a pseudomucinous polycystic cystomi with tubercular surface and thick viscous yellow content, 20x 15 cm in size. Inter nal envelope of the tumor is smooth. The capsule is sent on histological research • Postoperative diagnosis — pseudomucinous cystoma of the right ovary.

Ovarian resection (ovariotomy) The operation is performed in case of scleropolycystic ovarian syndrom t< normalize menstrual function and renewing of the reproductive ones. Stages of the operation: • disinfecting of skin • midline vertical or transverse incision of anterior abdominal wall • introduction of retractor and examination of genital organs • separation of intestinal bowels • ovarian fixation by fenestrated forceps • cutting off pathologically altered ovarian tissue by scalpel • putting of interrupted suture on ovary (an operating nurse gives suture needl and catgut) • lavage of abdominal cavity EXAMPLE OF THE OPERATIVE NOTES in the wedge-shaped ovarian resection Indications for the operation: scleropolycystic ovarian syndrom, primary infertility. Anesthetization: endotracheal anesthesia with Nitrous oxide and Oxygen mixture, and neuroleptanalgesia. Operation passing. An operative field has been processed by 2% Iodine spirit solution and by 70% Ethyl alcohol, edged by sterile surgical garb. Laparotomy by Pfannenshtiel. Before opening peritoneal cavity, the wound is edged by sterile serviettes and isolated from hypodermic fat. Bowels have been separated by serviettes. Findings during the examination of small pelvis are the following: uterus is not altered, normally developed, occupies central position in the cavity of small pelvis. Both ovaries are whitish in colour, their dimension exceeds normal considerably, surface is smooth. Both uterine tubes are without pathological changes. Both ovaries aren exteriorized out of abdominal cavity into the wound. About 2/3 of tissue from both ovaries are removed by wedge-shaped cut in hilus direction. Ovarian surfaces are jointed by interrupted catgut sutures. There was no bleeding. Lavage of abdominal cavity has been conducted. Calculation of serviettes and instruments — all are present. The incisional wound of anterior abdominal is sutured layer-by-layer: peritoneum and muscles — by continuous, aponeurosis — by interrupted catgut suture, skin and hypodermic fat — by interrupted cosmetic suture. Asepsis bandage has been dressed. There was no bleeding during the operation. Operation duration is 20 min. Name of the performed operation: wedge-shaped ovarian resection. Postoperative diagnosis — scleropolycystic ovarian syndrome.

No. 377-Hysterectomy for Benign Gynaecologic Indications

ABSTRACT Objective To assist physicians performing gynaecologic surgery in decision making surrounding hysterectomy for benign indications.

Intended Users Physicians, including gynaecologists, obstetricians, family physicians, general surgeons, emergency medicine specialists; nurses, including registered nurses and nurse practitioners; medical trainees, including medical students, residents, and fellows; and all other health care providers.

Target Population Adult women (18 years and older) who will undergo hysterectomy for benign gynaecologic indications.

Options The approach to hysterectomy and utility of concurrent surgical procedures are reviewed in this guideline.

Evidence For this guideline relevant studies were searched in the PubMed, Medline, and Cochrane Library databases. The following MeSH search terms and their variations for the last 5 years (2012-2017) were used: vaginal hysterectomy, laparoscopic hysterectomy, robotic hysterectomy, laparoscopically assisted vaginal hysterectomy, total laparoscopic hysterectomy, standard vaginal hysterectomy, and total vaginal hysterectomy. Validation methods The content and recommendations were drafted and agreed upon by the principal authors and members of the Committee. The Board of the Society of Obstetricians and Gynaecologists of Canada approved the final draft for publication. The quality of evidence was rated using the criteria described in the Grading of Recommendations Assessment, Development and Evaluation (GRADE) methodology framework ( Tables 1 and 2 ). The Summary of Findings is available upon request.

Benefits, Harms, and Costs Hysterectomy is common, yet surgical practice still varies widely among gynaecologic physicians. This guideline outlines preoperative and perioperative considerations to improve the quality of care for women undergoing benign gynaecologic surgery.

Guideline Update This Society of Obstetricians and Gynaecologists of Canada clinical practice guideline will be automatically reviewed 5 years after publication. However, authors can propose another review date if they feel that 5 years is too short/long based on their expert knowledge of the subject matter.

Sponsors This guideline was developed with resources funded by the Society of Obstetricians and Gynaecologists of Canada.

Summary Statements  1Technicity is defined as the proportion of performed by a minimally invasive route (laparoscopic, laparoscopic-assisted, and vaginal). Increased technicity index is associated with improved surgical quality and patient care (High).  2Minimally invasive approaches to hysterectomy are associated with fewer perioperative complications compared to laparotomy (High).  3Higher-volume hospitals and surgeons are more likely to have higher technicity and lower complication rates (High).  4Same-day discharge protocols following minimally invasive hysterectomy are cost-effective, do not increase complications or re-admission rates, and are associated with high patient satisfaction (Moderate).  5Urinary tract injuries are comparable among surgical approaches to hysterectomy (Moderate).  6Laparotomy or mini-laparotomy may be appropriate as an alternative approach in specific circumstances depending on patient factors, indication for surgery, and underlying pathology (Moderate).  7The risk of vaginal cuff dehiscence is rare and not related to the choice of suture material or route of closure (Moderate).  8Supracervical hysterectomy has not been shown to preserve sexual function, decrease pelvic organ prolapse, or reduce incidence of urinary tract injuries compared to total hysterectomy (Moderate).  9For women with uterine , preoperative medical treatment with leuprolide acetate or can reduce myoma size, decrease bleeding, and correct anemia. Risks and benefits of medical treatment should be discussed preoperatively (High).  10Mechanical bowel preparation is not routinely required prior to gynaecologic surgery for benign disease (High).  11Removal of normal ovaries at the time of hysterectomy decreases the risk of ovarian cancer but may be associated with health ramifications. Bilateral oophorectomy may lead to acute development of menopausal symptoms in premenopausal women and has not been shown to offer a survival benefit in the absence of genetic predisposition to ovarian cancer (High).  12Hysterectomy alone affects ovarian reserve (High).  13Opportunistic salpingectomy at the time of hysterectomy is expected to decrease the incidence of high-grade serous ovarian cancer (Low).  14There is no strong evidence to support routine uterosacral or vaginal vault suspension at the time of hysterectomy in patients without pelvic organ prolapse (Low). Recommendations  1Hysterectomy for benign indications should preferably be approached by either vaginal or laparoscopic routes (Strong, High).  2Vaginal hysterectomy is still considered the preferred route of hysterectomy, but laparoscopic hysterectomy is an appropriate alternative minimally invasive approach (Strong, Moderate).  3Correction of preoperative anemia (hemoglobin <120 g/L) is indicated to reduce morbidity and mortality in the perioperative period when elective surgery is planned (Strong, High).  4Preoperative antibiotic prophylaxis and measures to decrease risk of venous thromboembolism are recommended for all patients undergoing hysterectomy (Strong, High).  5Women should be counselled about the benefits and risks of removing the ovaries at the time of the hysterectomy. This should include discussion about the risk of ovarian cancer as well as the long-term health implications of earlier menopause linked to bilateral oophorectomy (Strong, Moderate).  6Opportunistic salpingectomy can be considered at the time of hysterectomy but the planned surgical approach should not be changed for this sole purpose (Strong, Low).  7Urinary tract injury is a known complication of hysterectomy, and clinicians should have a low threshold for further investigation in cases where injury is suspected. Surgeons performing hysterectomy should have access to diagnostic cystoscopy, individually or though consultation, to evaluate for bladder and ureteric integrity (Strong, Moderate).  8If patients with endometriosis are planning to undergo hysterectomy, full excision of local endometriosis should be performed concurrently (Strong, Moderate). V. Tests for self-assessment

1. The characteristic features of the hysteromyoma: a) Benign, hormonal-dependent tumor; b) Comes from (transversely) the striated muscular tissues; c) It is mainly diagnosed in women of the early reproductive age during the pubertal period; d) It is inclined to malignisation; e) All listed above.

2. A patient with fibromyoma of uterus sized up to 8-9 weeks of pregnancy consulted a gynaecologist about acute pain in the lower abdomen. Examination revealed pronounced positive symptoms of peritoneal irritation, high leukocytosis. Vaginal examination revealed that the uterus was enlarged corresponding to 9 weeks of pregnancy due to the fibromatous nodes, one of which was mobile and extremely painful. Appendages were not palpable. There were moderate mucous discharges. What is the optimal treatment tactics? A Urgent surgery (laparotomy) B Surveillance and spasmolytic therapy C Fractional diagnostic curettage of the uterine cavity D Surgical laparoscopy E Surveillance and antibacterial therapy

3. The genital endometriosis is: a) Expansion of endometrial like tissue beyond the usual localization of endometrium; b) A disorder that occurs on the background of hormonal and immune disorders; c) Pathology, which is mainly diagnosed in women of reproductive age; d) The pathological process is disposed to progression and recurrence; e) All listed above.

4. The most informative methods of diagnosis of the endometriosis are: a) Ultrasound; b) MRT; c) Determining the level of the tumor marker CA-125; d) Laparoscopy; e) Biopsy of heterotopias.

5. The most common symptoms of the internal endometriosis include: a) No symptoms; b) Algodysmenorrhea; c) Hypermenorrhea or polymenorrhea; d) Pain in the lumbar region; e) Cyclic bleeding from the rectum.

6. The external genital endometriosis includes all following forms, except of: a) The ovarian endometriosis; b) The uterine body endometriosis; c) The cervical endometriosis; d) The endometriosis of the ectopic space; e) The endometriosis of the interstitial part of the fallopian tubes.

7. The main clinical symptom of the endometriosis of the cervix is: a) Algodysmenorrhea; b) Menometrorthagia; c) Premenstrual and postmenstrual spotting; d) Infertility; e) None listed above.

8. An infertile woman may be given danazol (Danocrine) in order to: A Treat endometriosis B Stimulate her pituitary gland C Induce ovulation D Help her to relax prior to intercourse

9. A 55-year-old woman tells the nurse that she has started to experience pain when she and her husband have intercourse. The doctor would record that this woman is experiencing: A Dyspareunia B C Dysuria D Dyspnea

10. A 28-year-old woman has been diagnosed with endometriosis. She has been placed on a course of treatment with danazol (Danocrine). The woman exhibits understanding of this treatment when she says: A "I can experience a decrease in my breast size, oily skin, and hair growth on my face as a result of taking this ." B "Since this medication stops ovulation I do not need to use birth control." C "I will need to take this medication until 1 reach menopause." D "I will need to spray this medication into my nose twice a day."

11.A 32-year-old patient consulted a doctor about being inable to get pregnant for 5-6 years. 5 ago the primipregnancy ended in artificial abortion. After the vaginal examination and USI the patient was diagnosed with endometrioid cyst of the right ovary. What is the optimal treatment method? A Surgical laparoscopy B Anti-inflammatory therapy C Conservative therapy with estrogen-gestagenic drugs D Hormonal therapy with androgenic hormones E Sanatorium-and-spa treatment

12. A patient was admitted to the hospital with complaints of periodical pain in the lower part of abdomen that gets worse during menses, weakness, malaise, nervousness, dark bloody smears from vagina directly before and after menses. Bimanual examination revealed that uterus body is enlarged, appendages cannot be palpated, posterior fornix has tuberous surface. Laparoscopy revealed: ovaries, peritoneum of rectouterine pouch and pararectal fat have "cyanotic eyes". What is the most probable diagnosis? A Disseminated form of endometriosis B Polycystic ovaries C Chronic salpingitis D Tuberculosis of genital organs E Ovarian cystoma

Answers 1 – a, 2 – a, 3 - e; 4 - d; 5 - c; 6 - b; 7 – c; 8 – a; 9 – a; 10 – a; 11 – a; 12 – a

13.A pregnant 35-year-old patient is at highest risk for the concurrent development of which of the following malignancies? A. breast B. cervix C. ovary D. vagina E. colon

14.A patient is diagnosed with carcinoma of the breast. Which of the following is the most important prognostic factor in the treatment of this disease? A. axillary node metastases B. age at diagnosis C. size of tumor D. estrogen receptors on the tumor E. progesterone receptors on the tumor

15.A 43-year-old G2P2 comes to your office complaining of an intermittent right nipple discharge that is bloody. She reports that the discharge is spontaneous and not associated with any nipple pruritus, burning, or discomfort. On physical examination, you do not detect any dominant breast masses or adenopathy. There are no skin changes noted. Which of the following conditions is the most likely cause of this patient’s problem? A. duct ectasia B. intraductal papilloma C. breast cancer D. fibrocystic breast disease E. pituitary adenoma

16.A 20-year-old G0, LMP 1 week ago, presents to your gynecology clinic complaining of a mass in her left breast that she discovered during routine breast self- examination in the shower. When you perform a breast examination on her, you palpate a 2-cm firm, nontender mass in the upper inner quadrant of the left breast that is smooth, well-circumscribed, and mobile. You do not detect any skin changes, nipple discharge, or axillary lymphadenopathy. Which of the following is the most likely diagnosis? A. fibrocystic breast change B. fibroadenoma C. beast carcinoma D. fat necrosis E. cystosarcoma phyllodes

17. A 28-year-old woman, gravida 2, para 2, presents to your offi ce for a routine evaluation and you find an abnormality on her breast examination. She is asymptomatic, was unaware of the mass, and has no medical problems. There is no history of breast or ovarian cancer in her family. Her examination is notable for a 2-cm mass in her right breast that is smooth, mobile, and nontender. Your next step is: A. obtain an US of the mass B. reassure her that the mass is benign C. recommend vitamin E D. refer her to a breast surgeon for excision of the mass E. recommend a mammogram

18.A 60-year-old woman, gravida 3, para 2, SAb 1, presents to your clinic reporting brownish redcolored discharge from her left nipple. Her past medical history and medications, respectively, are as follows: diabetes—oral hypoglycemic; hypertension—angiotensin-converting enzyme inhibitor; major depression—fl uoxetine. She is also taking conjugated estrogen with medroxyprogesterone acetate daily. She is allergic to penicillin. She says her mother was diagnosed with ovarian cancer at the age of 71 years. What is the next best step in management? A. mammogram B. ultrasound C. referral to breast surgeon D. cessation of hormone replacement therapy E. fine-needle aspiration (FNA)

19. A 44-year-old premenopausal woman presents for evaluation of an area of thickening in the upper outer aspect of her left breast. She has a family history of breast cancer in her maternal grandmother and her mother, who tested positive for a BRCA1 mutation. The patient has not been tested. She has been aware of the left breast changes for 2 months, with some associated discomfort but no fluctuation with her periods. A screening mammogram performed within the last 3 to 4 months was negative. On physical examination there is glandular asymmetry in the left breast, but no discrete mass or regional adenopathy. What is the most appropriate immediate recommendation? A. left breast US B. bilateral breast MRI C. follow-up clinical examination in 3 months D. reassurance, i buprofen E. referral for genetic testing

Situational tasks Task 1 The patient of 44 years old was admitted to the hospital complaining of the severe pain in the lower abdomen and the sacral region. Before and during menstruation the pain increases sharply; urination and defecation are frequent; there is general weakness, in- creased nervousness. She suffered from the endometriosis for 5 years. Her first men- struation began at 14, it lasts 3—4 days, its cycle is 26—27 days; it's painful. In the history there was one pregnancy ended by its artificial interruption. 1. Make the diagnosis. 2. Identify the medical tactics.

Task 2 The patient of 29 years old complained of dark blood spotting from the genitals before and after menstruation. Vaginally: the cervix is cylindrical, the cervical ostium is closed, and the uterus is of normal size, mobile and painless. The uterine appendages are not defined. In specula: on the cervix were seen the small purplish-bluish cystic formations, from which dark blood flowing. 1. Make the diagnosis. 2. What methods should be applied to verify the diagnosis?

Task3 The patient of 37 years old was sent to the consultation. She complained of dysmeno- rrhea, dyspareunia, the rectal pain, premenstrual spotting and the pain in the spine. In palpation: sacro-uterine ligaments are sensitive, knotty; the uterus is fixed and dense. In laparoscopy: bluish-black formations are on the peritoneal surface. 1. Make the diagnosis. 2. Identify the medical tactics.

Task 4 The patient of 45 years old complained of the profuse discharge and the pain during menstruation, spotting bleeding from the genitals before and after menstruation. She considered being sick for 2 years; she didn't consult a doctor. In the history: one term childbirth; three medical abortions, the last was 2 years ago, complicated by hemato- metra, which was removed; then the control curettage of the walls of the uterus was carried out; after it the metroendometritis was developed. Objectively: the skin was pale pink, the heart rate was 72 per minute, BP was 125/80 mm Hg, the peripheral blood hemoglobin level was 100 g/l. The vaginal examination: the external genitals and the vagina were normal; the cervix was cylindrical, without erosions; the uterus was in retroflexion, enlarged to a size corresponding to 7—8 weeks of pregnancy, dense, partially mobile; its appendages on both sides weren't defined; the parametria were free; the discharges were mucous and light. 1. The most correct diagnosis is: a) The ovarian dysfunction of the perimenopausal period; b) The uterine fibroids; c) The endometrial cancer; d) The internal uterine endometriosis. 2. What methods should be used for the differential diagnosis?

Task 5 The patient of 46 years old visited the gynaecologist of the consultive dispensary with complaints of the long-term with profuse discharges, general weakness and decline of capacity. In anamnesis: two childbirths, five spontaneous abortions, the chronic inflammation of the uterine appendages. During the last 6 years with the pur- pose of contraception she used the endometrial contraceptive; the last was removed a year ago. She thinks she is ill about 6 months. She hasn't been consulted by the doctor yet. Objectively; the skin is pale-pink; the heartbeat is 78 beats per minute; the pulse's rhythmic; BP is 125/80 mm Hg; the level of hemoglobin is 80 g/l. Vaginal examination: the cervix of the uterus is not erosed; the uterine body is megascopic to the sizes, which is due to 10—11 weeks of pregnancy; the uterus is mobile, painless; the appendages on both sides aren't determined, painless; the vaults of the vagina are well expressed; and the discharges from the genital organs are mucous. 1. What gynecological diseases are characterized by the described above clinical pre- sentation? a) The hysteromyoma with the submucous location of one of the nodes. b) The internal endometriosis of the uterine body; c) The chronic endometritis; d) The dysfunctional uterine bleeding. 2. What additional methods of research would you use to confirm the diagnosis? a) The ultrasonic examination of the internal genital organs; b) The teleroentgeno-hysterosalpingography; c) The hysteroscopy with the separate diagnostic scraping off the mucous membrane of the cervical canal of the uterus and walls of the uterine cavity with the histological re- search of results of scraping; d) The laparoscope 3. The most credible reasons of the hyperpolymenorrhea are: a) Decline of the retractive activity of the myometrium; b) The relative hyperestrogeny; c) Increasing of the area of surface of the uterus; d) Violation of the coagulation of blood.

Answers Task 1. 1. Combined form of the endometriosis. 2.Performing the diagnostic and laparoscopy. Task 2. 1. The endometriosis of the cervix. 2.Biopsy. Task 3. 1. The retrocervical endometriosis. 2.Using of the medicative laparoscopy. Task 4. 1. d. The internal uterine endometriosis. 2.Ultrasound, the fractional curettage of the uterus, hysterosalpingography and determination of the tumor markers. Task 5. —1- a, 2-c, 3-b.